You are on page 1of 61

yoursmahboob.wordpress.

com

Practice Set 1
Numerical Ability 9. 2959.87 16.003 - 34.99 = ?
A. 160 B. 150 C. 140 D. 180 E. 170
Directions(1 5): What should come in place of the
question mark (?) in the following questions? 10. 424.99 23.95 8.05 = ?
1. 36865 + 12473 + 21045 - 44102 = ? A. 1300 B. 1225 C. 1325 D. 1275 E.
A. 114485 B. 28081 C. 26281 D. 114845 1375
E. None of these Directions(11 15): What should come in place
5+? 19 15 7 of the question mark (?) in the following number
2. =6
13 13 156 series?
A. 4 B. 4.5 C. 5 D. 5.5 E. None of
these 11. 513, 735, 1179, 1845, 2733, ?
A. 3713 B. 3843 C. 3863
D. 3793
3. 666.66 + 66.66 + 6.66 + 6 + 0.66 = ?
E. None of these
A. 746.64 B. 764.64 C. 766.64 D. 744.54
E. None of these 12. 9450, 2100, 600, 240, 160, ?
3 2 3
 
A. 80 B. 320 C. 40 D. 360 E. None of
4. 12 5 5
5 5 70 these
131 141 151 151
A. 1 B. 2 C. 1 D. 3 13. 6, 50, 490, 4890, 48890, ?
353 353 353 353
E. None of these A. 488890 B. 488900 C. 488990 D. 489898
1 1 1 1 E. None of these
5. 3 + 5 + ? + 2 = 11
2 5 4 3
11 13 23 29 14. 289, 361, 529, 841, 961, ?
A. B. C. D. E. None of
60 60 60 60 A. 2209 B. 2136 C. 1849 D. 1681
these E. 1369

Directions(6 10): What approximate value 15. 30, 34, 43, 59, 84, 120, ?
should come in place of the question mark (?) in A. 169 B. 148 C. 153 D. 176 E. None
the following questions? (Note: you are not ex- of these
pected to calculate the exact value).
16. Average of x observation is 30, average of x other
6. 2950 12.25 + 160 = ? observations is 33 and average of remaining x ob-
A. 440 B. 350 C. 380 D. 360 E. 400 servations is 42. Average of all the observations is
A. 34 B. 35 C. 36 D. Cannot be deter-
7. 25.05% of 2845 + 14.97 2399 = ? mined E. None of these
A. 36700 B. 36500 C. 35800 D. 35600
17. Due to reduction by 10% in the price of sugar a
E. 36200
person is able to buy 2 kg more for Rs. 480. Find
8. (1702 68) 136.05 = ? the reduced rate/kg of sugar.
A. 3500 B. 3550 C. 3450 D. 3400 A. Rs. 48 per kg B. Rs. 40 per kg C. Rs. 24
E. 3525 per kg D. Rs. 20 per kg E. None of these

1
yoursmahboob.wordpress.com
2 Practice Set 1

18. Rishi spends 40% of his salary on house rent, on re- 25. A, B, C enter into a partnership in the ratio 7 : 4
2 2 3
maining 10% spends on travel, on remaining 16 % 6
3 : . After 4 months, A increases his share 50%. If
spends on food and remaining is saved. If he saved 5
Rs. 2250 what amount he spent on food? the total profit at the end of one year be Rs. 21600
then Bs share in the profit is:
1
A. Rs. 433 B. Rs. 533 C. Rs. 430 D. Rs.
3 A. Rs. 2100 B. Rs. 2400 C. Rs. 3600
450 E. None of these D. Rs. 4000 E. None of these
19. The ages of two persons differ by 12 years. If 4
26. The amount of water (in ml) that should be added
years ago, the elder one be 3 times as old as the
to reduce 9 ml lotion, containing 50% alcohol, to a
younger one, find their present ages in years
lotion containing 30% alcohol is:
A. 12, 24 B. 10, 22 C. 15, 27 D. 17, 29
E. None of these A. 5 ml B. 4 ml C. 3 ml D. 6 ml E. 2
ml
20. The salaries of Ramu, Raju, Raghu are in the ratio
2 : 3 : 5. If the increments of 15%, 10% and 20% 27. Two trucks travel the same direction at the speed
are allowed respectively on their salaries, what will of 50 kmph and 60 kmph. Find distance when the
be their new ratio? time taken by both trucks has difference of 1 hour?
A. 3 : 3 : 10 B. 10 : 11 : 20 C. 23 : 33 : 60 A. 300 km B. 100 km C. 150 km D. Can-
D. 23 : 69 : 33 E. None of these not be determined E. None of these

21. The simple interest occurred on an amount of Rs. 28. In one hour, a boat goes 11 km/hr along the stream
22,500 at the end of four years is Rs. 10,800. What and 5 km/hr against the stream. The speed of the
would be the compound interest occurred on the boat in still water in km/hr is
same amount at the same rate at the end of 2
years? A. 3 km/hr B. 15 km/hr C. 8 km/hr D. 10
km/hr E. 9 km/hr
A. Rs. 16908 B. Rs. 5724 C. Rs. 28224
D. Rs. 8586 E. None of these 29. 8 men can complete a work in 14 days. They
22. If the rate of interest is 10% p.a, where interest started the work and after 2 days 2 men left. In
is compounded once every six months, what is the how many days will the work be completed by the
effective rate of interest? remaining men?
A. 12 days B. 9 days C. 16 days D. 6 days
A. 10% B. 20% C. 25% D. 10.25% E. None of these
E. Cannot be determined
30. A basket contains 4 red, 5 blue and 3 green mar-
23. Two concentric circles form a ring. The inner and
bles. If three marbles are picked at random, what
outer circumference of ring are 352/7 m and 528/7
is the probability that either all are green or all are
m respectively. Find the width of the ring.
red.
A. 6 m B. 6.5 m C. 5.5 m D. 4 m
E. None of these A. 7/44 B. 7/12 C. 5/12 D. 1/44
E. None of these
24. A cow is tethered in the middle of a field with a
14 feet rope. If the cow grazes 100 square feet per Directions(31 35): Each of the question and
day, then approximately what time will be taken two statements numbered I and II are given below
by the cow to graze the whole field? it. You have to decide whether the data provided
A. 10 days B. 14 days C. 6 days D. 7 days in the statements are sufficient to answer the ques-
E. 8 days tion. Read both statement and give answer.
yoursmahboob.wordpress.com
Practice Set 1 3

1. If the data in statement I alone is sufficient to II. The number obtained by interchanging the two
answer the question, while the data in state- digits of the number is lesser than the original num-
ment II alone is not sufficient to answer the ber by 18.
question. A. 1 B. 2 C. 3 D. 4 E. 5
2. If the data in statement II alone is sufficient to
answer the question, while the data in state-
ment I alone is not sufficient to answer the
Reasoning Ability
question.
3. If the data either in Statement I alone or in
Statement II alone is sufficient to answer the 36. How many such pairs of letters are there in the
question. word COURSE, each of which has as many letters
between them in the word (in both forward and
4. If the data in both the statements I and II are backward directions) as they have between them
not sufficient to answer the question. in the English alphabetical order?
5. If the data in both the statements I and II A. None B. One C. Two D. Three
together are necessary to answer the question. E. More than three

31. What is the area of the square? 37. In a certain SCENT is coded as UOFDT and
LASER is coded as SFTBM. In the same code
I. Measure of the diagonal of the square is 80 cm.
RETAIL will be coded as ......?
II. The perimeter of the square is equal to the cir- A. SFUBJM B. MJFUBS C. SFUJMB
cumference of a circle. D. KHBSDQ E. MJBUFS
A. 1 B. 2 C. 3 D. 4 E. 5
38. If each vowel of the word HOSPITAL is changed
32. What is Ashas present salary? to the next letter in the English alphabetical series
and each consonant is, changed to the previous let-
I. Her salary increases every year by 15 percent.
ter in the English alphabetical series and then the
II. She joined the organization seven years ago. alphabets thus formed are arranged in alphabetical
A. 1 B. 2 C. 3 D. 4 E. 5 order from left to right, which of the following will
be fifth from the right?
33. What is the rate of interest p.c.p.a? A. O B. J C. P D. K E. G
I. The compound interest acurred on an amount of 39. Which of the following will come in the place of
Rs. 1500 at the end of 2 years is Rs. 660. question mark(?)
II. An amount doubles itself in 5 years with simple AC EG JL ? WY
interest.
A. NP B. MO C. PR D. OP E. RT
A. 1 B. 2 C. 3 D. 4 E. 5
40. Among A, B, C, D, and E each scoring different
34. What is the total staff strength of the organization? marks in an examination. A scored less than only
I. 75% of the staff consists of male employees. B. D scored more than only C. Who amongst them
scored the third lowest marks?
II. The ratio of female to male employees in the
organization is 1 : 3 respectively. A. E B. D C. A D. Cannot be determined
E. None of these
A. 1 B. 2 C. 3 D. 4 E. 5
41. If K denotes x, B denotes , T denotes
35. What is the two digit number? and M denotes +, then 40 B 8 T 6 M 3 K 4 = ?
I. The sum of the two digits of the number is 8. A. 19 B. 11 C. -31 D. 23 E. None of
yoursmahboob.wordpress.com
4 Practice Set 1

these II. Some ponds are rivers


III. All rivers are trees
Direction(42 46): In each of the questions be-
A. Only I and III follows B. Only I and II
low are given three statements followed by three
follows C. Only II and III follows D. All I, II
conclusions numbered I, II and III. You have to
and III follows E. None of these
take the given statements to be true even if they
seem to be at variance from commonly known 45. Statements:
facts. Read all the conclusions and then decide
which of the given conclusions logically follows All desks are mirrors
from the given statements disregarding commonly Some mirrors are houses
known facts.
All houses are buildings
42. Statements: Conclusion:
Some buses are doors I. Some buildings are mirrors
Some doors are windows
II. Some houses are desks
All windows are gardens
III. Some buildings are desks
Conclusion:
A. None follows B. Only I follow C. Only II
I. Some gardens are buses follows D. Only III follows E. Only I and II
II. Some windows are buses follows
III. Some gardens are doors
46. Statements:
A. Only I follows B. Only II follows C. Only I
All roads are jugs
and II follow D. Only I and III follow E. None
of the above All jugs are pots

43. Statements: Some pots are cans


All lanterns are walls Conclusion:
No walls is brick I. Some cans are roads
Some bricks are chairs II. Some cans are jugs
Conclusion: III. Some pots are roads
I. Some chairs are lanterns A. Only I and II follows B. Only I and III
II. Some bricks are lanterns follows C. Only II and III follows D. All I, II
and III follows E. None of these
III. No chair is lantern
A. Only I follows B. Only II follows C. Only
either I or III follows D. Only III follows Directions(47 50): Following questions are
E. None of these based on five words givenbelow:
RAG FIN PUT LOW SUE
44. Statements:
(The new words formed after performing the men-
Some rivers are jungles
tioned operations may or may not necessarily be
All jungles are ponds meaningful English words)
All ponds are trees
47. In the given words, each of the consonants is
Conclusion: changed to next letter in the English alphabetical
I. Some trees are rivers series and each vowel is changed to the previous
yoursmahboob.wordpress.com
Practice Set 1 5

letter in the English alphabetical series, how many 53. What is the position of B with respect of E?
words thus formed are with no vowels appear? A. Third to the left B. Third to the right
A. None B. One C. Two D. Three C. Immediately to the right D. Second to the
E. More than three left E. Immediately to the left
48. If third alphabet in each of the words is changed 54. Four of the following five are alike in a certain way
to next alphabet in the English alphabetical order, based on their seating position in the above ar-
how many words having same or different vowels rangement and so form a group. Which is the one
will be formed? that does not belong to that group?
A. None B. One C. Two D. Three A. CB B. EA C. DF D. BC E. AC
E. Four
55. How many persons sit between C and F?
49. How many letters are there in the English alpha-
betical series between the second letter of the word A. None B. One C. Two D. Three
which is second from the right and the second let- E. Four
ter of the word which is second from the left of the
given words? Directions(56 58): Study the following ar-
A. Two B. Five C. Six D. Nine rangement carefully and answer the questions
E. Three given below:
185947125836592764529264123
50. If the positions of the first and the second alpha- 514283
bets of each of the words are interchanged, which of
the following will form a meaningful English word? 56. How many 5s are three in the above arrangement,
A. Both SUE and PUT B. FIN C. Both each of which is immediately followed by a digit
PUT and LOW D. LOW E. SUE which has a numerical value of less than five?
Directions(51 55): Study the following infor- A. None B. One C. Two D. Three
mation carefully and answer the given questions. E. More than three
A, B, C, D, E and F are sitting in a straight line 57. If all the even digits are deleted from the above
facing North. arrangement, which of the following will be seventh
(A) A sits third to the right of D. from the left end of the arrangement?
(B) Neither A nor D sits at any of the extreme A. 9 B. 5 C. 1 D. 3 E. 1
ends of the line.
58. How many such 1s are there in the above arrange-
(C) C sits second to left of E. ment, each which is immediately preceded by per-
(D) B sits second to right of F. fect square?
(E) B does not sit at an extreme end of the line. A. None B. One C. Two D. Three
E. More than three
51. If all the persons are made to sit in alphabetical or-
der from right to left, the positions of how many
are there in the original seating positions? Directions(59 60): Read the following infor-
mation carefully and answer the questions that fol-
A. None B. One C. Two D. Three
low.
E. Four
If A B means A is wife of B
52. Who sits at the extreme right hand corner of the
line? If A + B means A is brother of B

A. C B. E C. F D. Either C or E If A B means A is daughter of B


E. None of these If A - B means A is son of B
yoursmahboob.wordpress.com
6 Practice Set 1

59. Which of the following means T is father of P? P @ Q means, P is not smaller than Q.
A. P + Q R - T B. P R - T Q P # Q means, P is not greater than Q.
C. P + Q R T D. P - Q + T R P % Q means, P is neither greater nor equal to
E. None of these Q.
60. How is Y related to Z if Y V + W Z? P * Q means, P is neither smaller nor greater
A. Daughter B. Father-in-law C. Wife than Q.
D. Daughter-in-law E. Cannot be determined P $ Q means, P is neither smaller nor equal to
Q.
Directions(61 65): Study the following infor- Now according to the following statement, if they
mation carefully and answer the questions given are true, judge their Conclusion I, II, and III follow
below. definitely true.
M, V, K, D, T, J and R are seven friends studying
66. Statements
in different classes III, IV, V, VI, VII, VIII and
IX standards. Each of them has different favourite M % R, R # T, T * N
colours yellow, blue, red, white, black, green and Conclusions
violet. J likes red and studies in Class V. R likes
violet and studies in class III. M studies in Class I. N * R
VIII and does not like green and yellow. K likes II. N $ R
white and does not study in VII and in IV. D stud- III. N $ M
ies in VI and likes black. T does not study in IV
and who studies in class IV does not like green. A. All are true B. Either I or II is true C. Ei-
ther I or II and III is true D. Either I or III and
61. In which standard does V study? II is true E. None of these
A. IV B. IX C. VII D. Data inadequate
67. Statements
E. None of these
J # N, K @ N, T $ K
62. What is Ms favourite colour?
Conclusions
A. Red B. Yellow C. Green D. Blue
I. J % T
E. None of these
II. T $ N
63. In which standard does K study?
III. N @ J
A. III B. V C. IV D. VII E. None of
A. None is true B. I and II is true C. II and
these
III is true D. I and III is true E. All are true
64. What is Vs favourite colour?
68. Statements
A. Green B. Red C. Yellow D. Data
B * D, D @ H, H % F
inadequate E. None of these
Conclusions
65. In which standard does T study?
I. B * F
A. IV B. VII C. VIII D. IX E. None of
these II. B $ F
III. D $ F
Directions(66 70): In the following question A. None is true B. Either I or II is true C. Ei-
@, #, %, * and $ are used according to following ther I or II and III is true D. only III is true
meaning. E. All are true
yoursmahboob.wordpress.com
Practice Set 1 7

69. Statements dish. This angered him and he refused to have


T $ K, K # R, R * M breakfast. After finishing a few important duties
in the court, the king went to the dining hall for
Conclusions lunch. The food arrived piping hot. The king had
I. M * K hardly picked up the first morsel when he caught
II. M % T sight of a fly in the food. He left the table in
disgust and when the food was prepared again for
III. M $ K him, he found that he had lost his appetite. At
A. All are true B. Either I or III is true C. Ei- the dinner table, an ignorant minister started eat-
ther I or II is true D. Either II or III is true ing before the king had started. This offended the
E. None of these king. He stormed out of the dining hall and refused
to come back. As a result, the king could not eat
70. Statements anything at all that day. The enraged king was
V @ M, A $ M, R # V now convinced that Ramaya was really jinxed. He
Conclusions ordered that Ramaya be hanged. In those days the
kings word was the law. The soldiers had no op-
I. R # A tion but to take Ramaya to the gallows. While on
II.V @ A their way to the gallows, they met the court jester.
III. R $ M He heard the story from Ramaya. The jester felt
that it was an unfair decision and that none of
A. Only I is true B. Only II is true C. Only this was Ramayas fault. He came up with a plan
III is true D. None is true E. All are true to save Ramaya from being hanged. He whispered
something in Ramayas ears. Ramaya nodded be-
General English fore being whisked away.
It was mandatory to grant one last wish to the
Directions(71 80): Read the following passage person who was being hanged. When the soldiers
carefully and answer the questions given below it. asked Ramaya about his last wish, he asked them
Certain words / phrases have been printed in bold to convey a message to the king. He also requested
to help you locate them while answering some of that he be hanged only after hearing the kings
the questions. response. Tell the king that while it may be true
A man called Ramaya lived in a small village. De- that anyone who sees my face first thing in the
spite being a nice man, he did not have any friends. morning does not get anything to eat that day, its
It was well known in the town that anyone unfor- also true that if anyone sees the kings face first
tunate enough to look at him first thing in the thing in the morning, as I did, he has to lose his
morning would not be able to have food that day. life. So who is the greater jinx - the king or I ?"
That was why no one in the town liked encounter- said Ramaya.
ing Ramaya in the morning, if they could help it. On hearing the message, the king was stunned. He
As soon as the king heard this rumour he decided realised his foolishness and ordered his soldiers
to try it out himself. to stop the execution. He summoned Ramaya and
He summoned Ramaya to the palace and ordered offered him gifts. He also requested Ramaya not
him to sleep in a room adjacent to his bedcham- to say anything about the incident to the public.
ber. Ramaya had no choice but to follow the kings He was sure that the people of the village wouldnt
order. He feared that the rumour would come true. take well to the idea of having a jinxed person for a
The next morning, the first time the king did after ruler. Ramaya agreed. When the villagers saw Ra-
waking up, was to take a look at Ramaya. When maya returning with the gifts, they assumed that
the king reached the breakfast table, he saw that the rumour was false and that the king had given
the royal chef had forgotten to prepare his favourite the gifts to Ramaya as compensation. They were
yoursmahboob.wordpress.com
8 Practice Set 1

ashamed of their behaviour and swore that they jinxed


would never ignore Ramaya. D. One of the ministers had started eating be-
fore the king during dinner
71. Why had the king summoned Ramaya? E. None is true
A. He wanted to have Ramaya executed
B. He wanted all the villagers to realize their 75. Arrange the following incidents in a chronological
mistake and be friends with Ramaya order as they occurred in the passage.
C. He wanted to test the court jesters intelli-
gence 1. The court jester met Ramaya
D. He wanted to see Ramayas face, the first 2. Ramaya received gifts from the king
thing every morning
3. Ramaya slept at the palace
E. He wanted to verify whether the rumour
about Ramaya was true or false 4. The villagers realised their mistake

72. According to the rumour, what would happen if A. 1324 B. 3214 C. 3241 D. 3142
someone saw Ramayas face, first thing in the E. 3124
morning?
76. What was the kings request to Ramaya?
A. That person would not get a chance to eat, the
whole day A. That Ramaya must accept that he is unlucky
B. That person would be executed by the king B. That Ramaya must tell the villagers that the
C. That person would eat the whole day king is very kind
D. That person would receive gifts from the C. That Ramaya must not tell the villagers that
king a minister had insulted the king
E. That person would become an outcast in the D. That Ramaya must not tell the villagers
village about the incident
E. That Ramaya must not tell anyone about
73. When was the king convinced that Ramaya was the gifts he had received
jinxed?
Directions(77 78): Choose the word/group of
A. When he realised that he had not eaten the words which is most similar in the meaning to
whole day after he had seen Ramayas face the first the word/group of words printed in bold as used
thing in the morning in the passage.
B. When he realised that each time he saw Ra-
maya, he fell sick 77. Adjacent to
C. When he heard the rumour that anyone who A. Far away B. Beside C. Opposite to
saw Ramayas face first thing in the morning would D. Same as E. Below
not get a chance to eat the whole day
D. When he saw that the chef had not prepared 78. Caught sight of
his favourite dish on the day he had seen Ramayas
A. Searched for B. Forgot about C. Suddenly
face the first thing in the morning
noticed D. Held E. Nabbed
E. When the soldiers went to him with Ra-
mayas final wish Directions(79 80): Choose the word/group of
words which is most opposite in meaning to the
74. Which of the following is true according to the word/group of words printed in bold as used in
given passage? the passage.
A. Ramaya had many friends in the village
B. The king had dinner on the day he saw Ra- 79. Refused
mayas face first thing in the morning A. Unused B. Remembered C. Agreed
C. Ramaya told the villagers that the king was D. Invited E. Declined
yoursmahboob.wordpress.com
Practice Set 1 9

80. Ordered 5. On completion, your representative may


A. Requested B. Unplanned C. Punished kindly inform me of the annual insurance pre-
D. Silenced E. Explained mium
6. The rules state that the equipments have to
be insured against accidents and fire.
Directions(81 84): Read each sentence to find
out whether there is any grammatical error or id-
85. Which of the following should be the FOURTH
iomatic error in it. The error, if any, will be in
sentence after rearrangement?
one part of the sentence. The number of that part
is the answer. If there is No error, the answer is A. 1 B. 2 C. 3 D. 4 E. 5
(E). (Ignore errors of punctuation, if any)
86. Which of the following should be the FIFTH sen-
81. The Government has asked individuals (A)/ with tence after rearrangement?
income of over Rs.10 lakh to (B)/ electronic file A. 1 B. 2 C. 3 D. 4 E. 5
tax returns for the year 2011- 12, (C)/ something
which was optional till last year. (D)/ No error (E) 87. Which of the following should be the SECOND
sentence after rearrangement?
A. A B. B C. C D. D E. E
A. 1 B. 2 C. 3 D. 4 E. 5
82. Despite of curfew (A)/ in some areas, minor (B)/ Directions(88 90): Each question below has
communal incidents were reported (C)/ from dif- two blanks, each blank indicating that something
ferent areas of the walled city. (D)/ No error (E) has been omitted. Choose the set of words for each
A. A B. B C. C D. D E. E blank which best fits the meaning of the sentence
as a whole.
83. This comes (A)/ at a time (B)/ when fund alloca-
tion (C)/ is been doubled. (D)/ No error (E) 88. Drawing attention to the pitfalls of ....... solely on
Uranium as a fuel for nuclear reactors, Indian sci-
A. A B. B C. C D. D E. E entists warned that Uranium will not last for long
and thus research on Thorium as its ....... must be
84. As the prison will get (A)/ an official telephone fa-
revived.
cility soon, the prisoners (B)/ wont have to make
calls in discreet manner (C)/ through smuggled A. using, substitute B. believing, replacement
mobile phones. (D)/ No error (E) C. depending, reserve D. reckoning, option
E. relying, alternative
A. A B. B C. C D. D E. E
89. The move to allow dumping of Mercury ...... an
Directions(85 87): Rearrange the following six outcry from residents of the area who ...... that
sentences 1, 2, 3, 4, 5 and 6 in the proper sequence high levels of Mercury will affect their health and
to form a meaningful paragraph, then answer the destroy ecologically sensitive forest area.
questions given below them. A. resulted, insist B. provoked, fear C. in-
cited, determined D. activated, accept E. an-
1. The insurance policy has to be deposited with gered, believe
NABARD.
90. Even as the ...... elsewhere in the world are strug-
2. I intend to purchase a tractor and certain gling to come out of recession, Indian consumers
other agricultural equipments. are splurging on consumer goods and to ....... this
3. I shall therefore feel obliged if you send your growth, companies are investing heavily in various
representative to inspect the equipments sectors.
4. I shall be applying for a loan Rs.6 lakhs from A. economies, meet B. countries, inhibit
NABARD for that purpose C. governments, measure D. nations, inflict
yoursmahboob.wordpress.com
10 Practice Set 1

E. companies, counter 100. A. business B. troubles C. fun


D. responsibility E. worries
Directions(91 100): In the following passage
there are blanks, each of which has been numbered.
These numbers are printed below the passage and
against each five words are suggested, one of which
fits the blank appropriately. Find out the appropri-
ate word in each case. In the 1980s, Japan was re-
garded as a highly developed country. However in
recent years, Japans growth has (91) and the re-
cent earthquake and tsunami have (92) devastated
the country. As their nation (93) to cope with
the disaster, its youth are (94) to meet the chal-
lenges. Many young Japanese have become (95)
not only contributing essential items and money,
(96) also co-ordinating rescue efforts. Few people
(97) that Japans young people would bring about
its (98) after all nearly one in ten youth were
unemployed, many (99) only part-time and young
people were only supposed to have (100) on their
minds. Often it take a huge crisis to make a society
change to achieve its potential.

91. A. stop B. decrease C. drop D. declined


E. fell

92. A. too B. also C. not D. caused


E. complete

93. A. competes B. efforts C. need D. strug-


gling E. tries
Answers
94. A. together B. started C. rising
D. co-operative E. failing 1. C 2. C 3. A 4. C 5. C 6. E 7. A 8. D
9. B 10. D 11. B 12. B 13. A 14. E 15. A
95. A. knowledgeable B. heroes C. volunteer 16. D 17. C 18. D 19. B 20. C 21. B 22. D
D. jobless E. powerful 23. D 24. C 25. D 26. D 27. A 28. C 29. C
30. D 31. A 32. D 33. C 34. D 35. E 36. C
96. A. without B. even C. instead D. but
37. E 38. D 39. C 40. A 41. B 42. E 43. C
E. besides
44. B 45. B 46. E 47. D 48. C 49. B 50. E
97. A. thought B. dream C. realise D. know 51. C 52. B 53. A 54. D 55. C 56. C 57. D
E. perceived 58. B 59. C 60. D 61. A 62. D 63. E 64. C
65. B 66. C 67. E 68. A 69. B 70. D 71. E
98. A. changes B. downfall C. renewal 72. A 73. A 74. D 75. E 76. D 77. B 78. C
D. reforms E. independence 79. B 80. A 81. C 82. A 83. D 84. E 85. A
86. C 87. D 88. E 89. B 90. A 91. D 92. B
99. A. worked B. earnings C. employee 93. A 94. C 95. C 96. D 97. B 98. C 99. A
D. wages E. hire 100. D
yoursmahboob.wordpress.com
Practice Set 1 11

Detailed Answers 6. (E)


2950 12.25 + 160 = ?
Numerical Ability
2950
+ 160
1. (C) 12
= 245 + 160 = 405 400
36865 + 12473 + 21045 - 44102 = 26281
7. (A)
2. (C)
25.05% of 2845 + 14.97 2399 = ?
5+? 19 15 7
=6 25
13 13 156 = 2845 + 15 2399
100
5 + 19x 22 = 25 28.45 + 15 2399
=6
169 156
= 25 28 + 35985
19x 17
=6 = 700 + 35985
13
19x = 78 + 17 = 36685 36700
19x = 95
8. (D)
x=5
(1702 68) 136.05 = ?
3. (A) = 25 136
666.66 + 66.66 + 6.66 + 6 + 0.66 = 746.64 = 3400

4. (C) 9. (B)
3 2 3
 
12 5 5 2959.87 16.003 - 34.99 = ?
5 5 70 2960
- 35
1 3 36 353 16
=7 5 =
5 70 5 70 = 185 - 35 = 150
36 70 504
= = 10. (D)
5 353 353
151 424.99 23.95 8.05 = ?
=1
353 24
425
8
5. (C)
425 3 = 1275
1 1 1 1
3 + 5 + x + 2 = 11
2 5 4 3 11. (B)
7 26 9 34 513 735 1179 1845 2733 3843
+ +x+ =
2 5 4 3
70 + 104 + 20x + 45 34 222 444 666 888 1110
=
20 3
219 34 12. (B)
+x=
20 3 9450 2 = 18900/9 = 2100
34 219
x= - 2100 2 = 4200/7 = 600
3 20
680 657 600 2 = 1200/5 = 240
x=
60 240 2 = 480/3 = 160
23
x= 160 2 = 320/1 = 320
60
yoursmahboob.wordpress.com
12 Practice Set 1

13. (A) Remaining salary = 3000 - 300 = 2700


6, 50, 490, 4890, 48890, ? 2
Amount spent on food = 16 % of 2700 = Rs.
3
(6 10) - 10 = 50 450
(50 10) - 10 = 490
19. (B)
(490 10) - 10 = 4890
Let the ratio before 4 years be 3 : 1
(4890 10) - 10 = 48890
Difference between ages = 12 years
(48890 10) - 10 = 488890
3 1
14. (E) 2x = 12
(17)2 = 289 x=6
(19)2 = 361 The ages before 4 years are 18 & 6
(23)2 = 529 Present ages are 22 & 10
(29)2 = 841
20. (C)
(31)2 = 961
Ratio of salaries of Ram, Raju and Raghu is 2 : 3
(37)2 = 1369 : 5
15. (A) Increments are = 15%, 10%, 20%

30 34 43 59 84 120 169 Let their salaries be 2x, 3x and 5x


115 110 120
2x : 3x : 5x
4 9 16 25 36 49 100 100 100
23 : 33 : 60
16. (D)
21. (B)
Cannot be determined
P = 22500, n = 4, S.I = 10800
17. (C)
100 22500
Reduction in price of sugar = 10%
4r 10800
10% of 480 = 48
r = 12%
From that amount a person able to buy 2 kg.
C.I be,
2 kg = 48 12
22500 = 2700 2 = 5400
1 kg = 24 100
12
18. (D) 2700 = 324 1 = 324
100
Let salary be x C.I is 5724
40 10 60 2 54
   
x+ x + 16 x + 22. (D)
100 100 100 3 100 100
2250 = x r = 10%
4 6 50 54
x+ x+ x + 2250 = x Interest is compounded every 6 months
10 100 300 100
x = 5000 1 year = 2 half year
40 10
40% spent on house rent = 5000 = Rs. 2000 r% for half year = = 5%
100 2
Remaining salary = 5000 - 2000 = 3000 Let amount be 100
10 5
10% spent on travel = 3000 = Rs. 300 100 = 5 2 = 10
100 100
yoursmahboob.wordpress.com
Practice Set 1 13

5 water = 4.5 ml
5 = 0.25 1 = 0.25
100
alcohol = 30% , water = 70%
Effective interest rate is 10.25%
4.5 + x 70
=
23. (D) 9+x 100
Circumference of the circle = 2r x = 6 ml
Inner circle circumference (r1 ) Amount of water that should be added = 6 ml
2r1 = 352/7 m 27. (A)
Circumference of outer circle of (r2 ) Speed of truck 1 = 50 km/hr
2r2 = 528/7 m Speed of truck 2 = 60 km/hr
22 352 22 528
2 r1 = ;2 r2 = Let the distance be x
7 7 7 7
r1 = 8, r2 = 12 Difference between the time taken by two trucks =
1 hour
Width = r2 - r1 = 12 - 8 = 4 m x x
=1
24. (C) 50 60
6x 5x
Length of rope = 14 feet (Radius of circle) =1
300
22 Therefore, the distance covered = 300 km
Area of circle = r 2 = 14 14
7
= 616 sq.feet 28. (C)
Cow takes 1 day to graze 100 sq.ft Speed of boat along the stream a = 11 km/hr
616 Speed of boat against stream b = 5 km/hr
If takes = 6.16 6 days to graze the field
100 1
Speed of boat in still water = (a + b)
25. (D) 2
7 4 6 1
Ratio of A, B, C is : : = (11 + 5) km/hr
2 3 5 2
7 4 6 16
: : = km/hr
2 3 5 2
105 : 40 : 36 = 8 km/hr
Let x be share 29. (C)
3
(105x 4) + (105x 8) : 40x 12 : 36x 8 men can do a work in 14 days.
2
12 1
1 mans 1 day efficiency =
105x 4 + 105x 12 : 40x 12 : 36x 12 8 14
105x 16 : 40x 12 : 36x 12 8 men worked for 2 days
1 1
35 : 10 : 9 Work completed = 2=
14 7
Profit at end of 1 year = Rs. 21600 1
Remaining work to be completed is 1 -
10 7
Bs share = 21600
54 6
=
= Rs. 4000 7
Number of men present after 2 days = 6
26. (D)
1 6
9 ml lotion containing 50% alcohol 6 x =
8 14 7
50% of alcohol = 4.5 ml of lotion x = 16 days
yoursmahboob.wordpress.com
14 Practice Set 1

30. (D) (120)2 = (100 + r)2


4R, 5B, 3C 120 = 100 + r
4C3 + 3C3 r = 20%
=
12C3
II. Amount doubles itself in 5 years with C.I
432 321
+ Amount = 2p
= 1 2 3 123
12 11 10 S.I = 2p - p = p
123
100 p
4+1 1
= = 5r p
220 44
31. (A) r = 20%
I. Diagonal of square = 80 cm 34. (D)

2a2 = 80 Total staff strength = ?
a = 56.577 75
I. 75% are male =
Area = a2 = 3201.30 100
II. Ratio of Male to Female is 1 : 3
It is sufficient to answer.
Let x be strength
II. Perimeter of square is equal to the circumfer-
ence of circle. x & 3x are M & F
75 25 75
It is not sufficient to answer. M= 1,F= 3=
100 100 100
Statement I alone is sufficient to answer. Total strength = cannot be determined.

32. (D) 35. (E)


To find Ashas present salary Two digit number = ?
I. Increase in salary every year = 15% I. Sum of the 2 digit number = 8
II. She joined seven years ago II. difference between original & interchanged num-
It does not have any known values. Both state- ber = 18
ments are not sufficient to answer the question. The number be = 53

33. (C)
Rate of interest = ? Reasoning Ability
I. p = 1500; n = 2; C.I = 660

r n
  36. (C)
C.I = p 1 + 1
100
" 2 # C O U R S E
r
660 = 1500 1+ 1
100

r
 TWO
660 = 1500 1+ 2 - 1500
100 37. (E)
100 + r 100 + r
  
2160 = 1500 S C E N T
100 100
2 7200 = (100 + r)2
14400 = (120)2 U O F D T
yoursmahboob.wordpress.com
Practice Set 1 15

R E T A I L 44. (B)

M J B U F S R J P T

38. (D)
H O S P I T A L
I - Follows
II - Follows
G P R O J S B K
III - Does not follow
Alphabetical order
Only I & II follow
B G J K O P R S
45. (B)
39. (C)
AC EG JL ? WY
M
D H B
= PR

40. (A) I - Follows


B>A>E>D>C II - Does not follow
=E III - Does not follow
41. (B) Only I follows
40 8 - 6 + 3 4 = ? 46. (E)
5 - 6 + 12 = 11
P
42. (E) C
R J
B D W G
I - Does not follow
I - Does not follow II - Does not follow
II - Does not follow III - Follows
III - Follows Only III follows
Only III follows 47. (D)
43. (C) Three

48. (C)
L W B C Two

I - Does not follow 49. (B)

II - Does not follow Five

III - Does not follow 50. (E)


By merging I & III, only either I or III follows SUE
yoursmahboob.wordpress.com
16 Practice Set 1

Directions(51 55): 62. (D)


F D B C A E Blue

63. (E)
51. (C) None of these
Two 64. (C)
52. (B) Yellow
E 65. (B)
53. (A) VIIth
Third to the left
Direction(66 70):
54. (D)
BC
% # *
55. (C) < =
Two > =
$ @ *
56. (C)
Two 66. (C)
M%R#TN
57. (D)
I) N R - does not follow
3
II) N $ R - does not follow
58. (B)
III) N $ M - follows
One
I & II can be merged
59. (C) Either I or II and III is true
P+QRT
67. (E)
60. (D) J#N#K%T
Daughter-in-Law I) J % T - follows
Directions(61 65): II) T $ N - follows
Friend Class Colour III) N @ J - follows
M VIII Blue
V IV Yellow All are true
K IX White
68. (A)
D VI Black
T VII Green BD@H%F
J V Red I) B F - does not follow
R III Violet
II) B $ F - does not follow
61. (A) III) D $ F - does not follow
IVth None is true
yoursmahboob.wordpress.com
Practice Set 1 17

69. (B)
T$K#RM
I) M K - does not follow
II) M % T - does not follow
III) M $ K - does not follow
I & III can be merged
Either I or III is true

70. (D)
R#V@M%A
I) R # A - does not follow
II) V @ A - does not follow
III) R $ M - does not follow
None is true
yoursmahboob.wordpress.com

Practice Set 2
Reasoning Ability Direction(6 8): Study the following arrange-
ment carefully and answer the questions given be-
1. Pointing towards a boy, Veena said, "He is the son low:
of only son of my grandfather". How is that boy BCM@N&T2Y6S?Q$7*W#Z3UE
related to Veena? %A4
A. Uncle B. Brother C. Cousin D. Data
6. How many symbols are there in the above series
inadequate E. None of these
each of which is immediately preceded and also im-
mediately followed by a vowel?
2. Introducing Reeta, Sarika said, "she is the only
daughter of my fathers only daughter". How is A. None B. One C. Two D. Three
Sarika related to Reeta? E. More than three

A. Aunt B. Niece C. Cousin D. Data 7. If all the vowels are dropped from the above series,
inadequate E. None of these which of the following would be the eighth element
to the right of the thirteenth element from the left
Directions(3 5): Study the following informa-
end?
tion to answer the given questions:
In a certain code, A. 4 B. B C. C D. % E. None of these
always create new ideas is written as ba ri
8. If each symbols is first converted into a numeral
sha gi,
and then all the numerals are converted into En-
ideas and new thoughts is written as fa gi ma
glish letters, how many English letters will be there
ri,
in the above arrangement of elements?
create thoughts and insights is written as ma
jo ba fa and A. 7 B. 12 C. 13 D. 25 E. None of
new and better solutions is written as ki ri these
to fa
9. In a certain code, ROAD is written as URDG. How
3. What is the code for ideas? is SWAN written in that code?
A. sha B. ba C. gi D. ma E. Cannot be A. VXDQ B. VZDQ C. VZCP D. UXDQ
determined E. None of these

4. fa lo ba could be a code for which of the following? 10. In a certain code, FAVOUR is written as EBUPTS.
A. thoughts and action B. create and innovate How is DANGER written in that code?
C. ideas and thoughts D. create new solutions A. CBFFDS B. CBMHDS C. EBFHDS
E. always better ideas D. EBHHFS E. None of these

5. Which of the following may represent insights al- 11. If you are facing north-east and move 10 m forward,
ways better? turn left and move 7.5 m, then you are
A. jo ki to B. ki to ri C. sha jo ri D. to A. North of your initial position
sha jo E. sha to ba B. South of your initial position

1
yoursmahboob.wordpress.com
2 Practice Set 2

C. East of your initial position 15. Statement: Despite good economic progress of
D. 12 m from your initial position the country, significant number of undernourished
E. Both C and D children has been observed in the rural parts of the
country.
12. A child is looking for his father. He went 90 metres Courses of action:
in the east before turning to his right. He went 20 I. Govt should increase Wealth Tax/ Income tax
metres before turning to his right again to look and use that money for upliftment of the deprived
for his father at his uncles place 30 metres from class.
his point. His father was not there. From there, II. Govt should introduce schemes like free meals
he went 100 metres to his north before meeting in primary schools and make primary education
his father in a street.How far did the son meet his compulsory.
father from the starting point?
A. Option A B. Option B C. Option C
A. 80 m B. 100 m C. 140 m D. 180 m D. Option D E. Option E
E. 260 m
16. Statement: Launching of new brands of four-
13. If it is possible to make only one meaningful word wheelers is adding to the traffic congestion in the
with the first, fifth, ninth and the tenth letters of metro cities.
the word AUTO MOBILE, which would be the Courses of action:
second letter of the word from the right ? If more I. Pubic should be encouraged to share their pri-
than one such word can be formed, give Y as the vate vehicles while travelling to their work places.
answer,if no such word can be formed, give Z as II. Govt. should levy heavy taxes on motor cars in
your answer. metro cities.
A. Y B. L C. A D. E E. Z
A. Option A B. Option B C. Option C
Directions(14 16): In each question below is
D. Option D E. Option E
given statement followed by two courses of action
numbered I and II. A course of action is a step Directions(17 20): Read the following infor-
or administrative decision to be taken for improve- mation carefully and answer the question given be-
ment, follow-up or further action in regard to the low.
problem, policy, etc. On the basis of the infor-
mation given in the statement, you have to assume 1. A, B, C, D, E, F and G are sitting around a
everything in the statement to be true, then decide circle and are facing the centre.
which of the suggested courses of action logically 2. G is second to the left of C, who is to the
follows for pursuing. immediate left of F.
Give answer (A) if only I follows. 3. A is third to the left of E
Give answer (B) if only II follows
Give answer (C) if either I or II follows. 4. B is between D and E
Give answer (D) if neither I nor II follows.
17. Which of the following is false?
Give answer (E) if both I and II follow.
A. A is 4th to the right of E
14. Statement: Cases of road accidents are increasing B. G is to the immediate right of D
constantly, particularly in the urban areas. C. F is 3rd to the right of D
Courses of action: D. B is to the immediate left of D
I. Transport authorities in the urban should impose E. None of these
stringent norms for maintenance of vehicles.
II. Traffic police should severely punish those found 18. Which of the following pairs has the 1st person
to be violating traffic rules. sitting to the immediate left of the 2nd person?
A. Option A B. Option B C. Option C A. BE B. CA C. GD D. DG E. None
D. Option D E. Option E of these
yoursmahboob.wordpress.com
Practice Set 2 3

19. Which of the following has the middle person sit- 23. What will be the last step for the following input?
ting between the remaining two? Input: 138 63 49 93 89 122 32 71
A. FCE B. EFB C. DEB D. GDA A. 32 49 63 71 89 93 122 138
E. None of these B. 32 49 63 71 93 89 122 138
C. 32 49 71 63 89 93 122 138
20. Which of the following is the position of F? D. Cant be determined
A. 4th to the right of D E. None of these
B. To the immediate left of C
24. How many step will be required for getting the final
C. Between A and E
output for the following input?
D. To the immediate right of A
E. None of these Input: 101 85 66 49 73 39 142 25 115 74
A. 5 B. 7 C. 6 D. 8 E. None of these
Directions(21 25): Study the following infor- 25. What will be the third step for the following input?
mation carefully to answer these questions.
Input: 67 23 58 159 46 123 79
A number sorting machine when given an input of A. 23 67 58 46 159 123 74
numbers, rearranges them in a particular manner B. 23 67 58 46 123 79 159
step-by-step as indicated below is an illustration of C. 23 46 67 58 123 79 159
this arrangement. D. 23 46 67 58 79 123 159
Input: 39 121 48 18 76 112 14 45 63 96 E. Cant be determined
Step I: 14 39 121 48 18 76 112 45 63 96
Directions(26 30): Read the following infor-
Step II: 14 39 48 18 76 112 45 63 96 121 mation and answer the questions given below it.
Step III: 14 18 39 48 76 112 45 63 96 121 Alka is older than Mala. Gopal is older than Mala
Step IV: 14 18 39 48 76 45 63 96 112 121 but younger than Alka. Kapil is younger than Ram
and Mala. Mala is older than Ram.
Step V: 14 18 39 45 48 76 63 96 112 121
26. Whose age is between Gopal and Ram?
Step VI: 14 18 39 45 48 63 76 96 112 121 and step
VI is the last step for this input. A. Mala B. Kapil C. Alka D. None of
these
21. What will be step III for the following input?
27. Whose age is between Mala and Kapil?
Input: 68 182 39 93 129 46 21 58 A. Gopal B. Ram C. Alka D. None of
A. 21 39 68 129 93 46 58 182 these
B. 21 39 68 93 129 46 58 182
28. Whose age is exactly in the middle of all the five?
C. 21 68 39 93 129 46 58 182
D. Cant be determined E. None of these A. Mala B. Gopal C. Ram D. Alka
29. Who is the eldest?
22. Given below is the fifth step of an input. What
will be the third step? A. Alka B. Mala C. Kapil D. Gopal

Step V: 17 32 43 82 69 93 49 56 99 106 30. Who is the youngest?


A. 17 32 82 43 69 93 49 56 99 106 A. Mala B. Ram C. Alka D. Kapil
B. 17 32 82 69 43 93 49 56 99 106
C. 17 32 82 69 93 43 49 56 99 106 Directions(31 35): Which conclusion is true
D. 17 32 82 69 43 93 56 49 99 106 among the following options for the given state-
E. Cant be determined ments.
yoursmahboob.wordpress.com
4 Practice Set 2

31. Statements: C. All pitbulls are dogs


i) All smart asses are dumb asses D. None of these

ii) Some bad asses are smart asses


Conclusions: Numerical Ability
A. All dumb asses are bad asses Directions(36 40): What should come in place
B. Some dumb asses are bad asses of question mark (?) in the following questions?
C. No bad asses are dumb asses
D. Some bad asses are not dumb asses 1
36. 8500 + (1600 ?) of = 8501
E. None of these 5
A. 310 B. 315 C. 320 D. 325 E. None
32. Statements: of these
i) All cities have houses 1
37. 377 13 29 2 =?
4
ii) All lands have cities 1 1 1 1
A. B. C. D. E. None of these
Conclusions: 8 6 4 3
A. All houses have lands 38. 5 [5 {5 5(5 + 5)}] = ?
B. All lands have houses A. 0 B. 1 C. 10 D. 50 E. None of these
C. Not all houses have lands
D. None of these 39. 175 11 50 3 (75 50) = ?
A. 1919 B. 1918 C. 1912 D. 1911
33. Statements: E. None of these
i) All kitchens have closets 40. 1 [1 + 1 {1 + 1 (1 1)}] = ?
ii) All closets have shelves 5 4 2 1
A. B. C. D. E. None of these
3 3 3 3
Conclusions:
A. Some kitchens have no shelves Directions(41 45): What approximate value
B. All shelves have kitchens should come in place of the question mark (?) in
C. All kitchens have shelves the following questions? (Note: you are not ex-
D. None of these pected to calculate the exact value).

34. Statements: 41. (186 270.99) 40 = ?


i) All ambulances are life savers A. 1160 B. 1200 C. 1300 D. 1180
ii) No ambulances are bumper cars E. 1260

Conclusions: 42. 4780 296 (23)2 = ?


A. No life savers are bumper cars A. 9870 B. 6760 C. 7590 D. 3430
B. No bumper cars are life savers E. 8540
C. Some life savers are not bumpers cars
D. None of these 43. 175 28 275 27.98 = ?
A. 470 B. 500 C. 600 D. 450 E. 700
35. Statements:
i) All dogs are brown 44. 624.98 + 729.95 = ?

ii) All pitbulls are brown A. 58 B. 56 C. 52 D. 63 E. 61

Conclusions: 45. 69.008% of 699.998 + 32.99% of 399.999 = ?


A. Some pitbulls are dogs A. 615 B. 645 C. 675 D. 715 E. 725
B. Some pitbulls are not dogs
yoursmahboob.wordpress.com
Practice Set 2 5

Directions(46 50): What should come in place 55. Peter having a bag of coins of 25p, 10p, 5p in the
of the question mark (?) in the following number ratio of 2 : 3 : 4. If there are Rs. 45 in all how
series? many 5p coins are there?

46. 24, 36, 72, 180, 540, 1890, ? A. 200 B. 90 C. 45 D. 180 E. None of
A. 4320 B. 3640 C. 7560 D. 5610 these
E. None of these
56. An amount of money invested on simple interest
47. 8, 31, 122, 485, 1936, 7739, ? becomes 5 times of itself in 16 years. What is the
A. 30460 B. 30720 C. 30840 D. 30950 rate of interest per annum?
E. None of these A. 15% B. 20% C. 25% D. 30%
E. None of these
48. 234, 448, 876, 1732 ?
A. 3444 B. 3222 C. 3333 D. 3555 57. A certain sum of money amounts to Rs. 1008 in 2
E. None of these 1
years and to Rs. 1164 in 3 years. Find the sum
2
49. 4248, 2112, 1074, 513, 286.5, ? and rate of interest?
A. 136.25 B. 161.25 C. 175.25 D. 112.25
A. 700, 13% B. 700, 15% C. 800, 13%
E. 107.25
D. 800, 15% E. None of these
50. 1140, 1148, 1175, 1239, 1364, ?
58. Arun walked diagonally across a square plot. Ap-
A. 1379 B. 1394 C. 1452 D. 1463
proximately what was the percent saved by not
E. None of these
walking the edges?
51. The average of the daily incomes of A, B and C is A. 50% B. 29.5% C. 35% D. 41%
Rs. 250. If B earns Rs. 30 more than C and A E. Cannot be determined
earns double of C. What is the daily income of C?
A. Rs. 195 B. Rs. 190 C. Rs. 180 D. Rs. 59. Three partners shared the profit in a business in
185 E. None of these the ratio 5 : 7 : 8. They had partnered for 14
months, 8 months and 7 months respectively what
52. Sandeep purchases a bike for Rs. 4700 and spends was the ratio of their investment?
Rs. 800 on its repair. If he sells the bike for Rs. A. 5 : 7 : 8 B. 20 : 49 : 64 C. 38 : 28 : 21
5800, his gain percent is: D. 25 : 30 : 42 E. None of these
4 5
A. 4 % B. 5 % C. 10% D. 12%
7 11 60. How many kg of pure salt must be added to 30 kg
E. None of these of a 2% solution of salt and water to increase it to
53. The population of a town increased from 175000 to a 10% solution?
262500 in a decade. What is the average percent 2 1
A. 2 kg B. 15 kg C. 3 kg D. 14 kg
increase of population per year? 3 3
E. None of these
A. 4% B. 6% C. 5% D. 50% E. None
of these 61. A boy goes to school at a speed of 3 kmph and
returns to the village at a speed of 2 kmph. If he
54. The present age of a mother is 2 years more than takes 5 hrs in all, what is the distance between the
three times the age of her son. Three years hence, village and the school?
the mothers age will be 8 years more than twice
A. 2 km B. 3 km C. 5 km D. 6 km
the age of the son. Find the present age of the
E. 4 km
mother.
A. 33 years B. 29 years C. 44 years D. 36 62. How many 4 digit numbers can be formed from the
years digits 1, 2, 3, 4, 5, 7, 9 which are divisible by 5 and
yoursmahboob.wordpress.com
6 Practice Set 2

none of the digits is repeated? 68. Total how many students are there in the Institute
A. 50 B. 90 C. 120 D. 15 E. None of of Arts, Commerce and Science?
these I. 20% of the students study Science.
II. Number of students studying Arts and Com-
63. A can do a piece of work in 80 days. He works at it
merce are in the ratio of 3 : 5 respectively.
for 10 days and then B alone finishes the remaining
work in 42 days. The two together could complete III. Number of students studying Commerce is
the work in how many days? more than the number of students studying Sci-
ence by 375.
A. 20 days B. 18 days C. 30 days D. 48
days E. None of these A. II and III B. III and either I or II C. Any
two of the three D. Cant be determined
64. A man can row upstream at 8 km/h and down- E. All of these
stream at 12 km/h. Find the mans rate in still
69. What is the cost of flooring the rectangular hall?
water.
I. Perimeter of the hall is 76 m.
A. 6 B. 8 C. 10 D. 12 E. None of these
II. Area of the hall is 336 sq.m.
65. A box contains 5 green, 4 yellow and 3 white mar- III. Cost of flooring per square metre is Rs. 550.
bles. Three marbles are drawn at random. What
is the probability that they are not of the same A. I and III B. II and III C. Any two of the
colour? three D. All of these E. None of these

A. 3/44 B. 3/55 C. 52/55 D. 41/44 70. In how many days can the work be completed by
E. None of these A and B together?
Directions(66 70): In each of these questions, I. A alone can complete the work in 8 days.
a question is followed by information in three state- II. If A alone works for 5 days and B alone works
ments. You have to study the question along with for 6 days, the work gets completed.
the information given in the statements and de-
III. B alone can complete the work in 16 days.
cide the information in which of the statement (s)
is necessary and sufficient to answer the question. A. I and II B. II and either I or III C. Either
II or I and III D. II and III E. None of these
66. What is Sudhas present salary?
I. The salary increases every year by 15%. General English
II. Her salary at the time of joining was Rs. 10000.
Directions(71 80): Read the following passage
III. She had joined exactly 5 years ago.
carefully and answer the questions given below it.
A. II and III B. I and II C. I and III D. All Certain words/phrases have been printed in bold
of these E. None of these to help you locate them while answering some of
the questions.
67. What was the amount of profit earned?
Renewable Energy Technologies (RETs) have long
I. 10% discount was offered on the labelled price. been recognised for their potential as environ-
II. The labelled price is Rs. 600. ment friendly, versatile and sustainable energy al-
ternatives for rural areas of India. However, de-
III. Selling price was more that the cost price by spite efforts by the Ministry of Non-conventional
20%. Energy Sources (MNES) and the India Renewable
A. I and either II or III B. Any two of the three Energy Development Agency (IREDA), RETs have
C. Either I or II and III D. Cant be determined not yet succeeded as a major alternative source of
E. All of these energy in rural India. The programmes of MNES
yoursmahboob.wordpress.com
Practice Set 2 7

and IREDA designed to support small-scale dis- of renewable energy resources. As extensive time
tributed systems have relied on heavily subsidized and resources are necessary to catalogue the site
credit, technology training and consumer aware- specific resource availability before financing can
ness activities to stimulate the market for end-user be considered, the technical and managerial know-
finance for renewable energy systems for domestic how requirements and associated costs that occur
use and a tiered set of capital and interest rate sub- prior to the project often represent a much larger
sidies for water pumping in the agriculture sector. percentage of project costs than in the case of con-
While end-users in some rural areas now have ac- ventional power projects.
cess to solar-powered lanterns or lights and biogas
Third, entrepreneurs developing small-scale renew-
systems for their home, and agricultural operations
able energy projects face the same business and fi-
are taking advantage of remarkable capital subsi-
nancial risks associated with any enterprise. The
dies for solar energy to use for water pumping, the
viability of any enterprise depends on its ability to
use of heavy capital and interest rate subsidies, and
build a business based on solid concepts and com-
the focus on domestic use rather than on matching
petencies in sound contexts that mitigate those
renewable energy technology applications with in-
risks.
come enhancement opportunities have tied the suc-
cess of these programmes to government budgets Considering the aforementioned characteristics,
and political cycles limiting both the breadth and the key barriers to development of small-scale re-
depth of development and penetration of projects newable energy projects are fourfold. First, rural
that harness renewable energy resources. entrepreneurs do not usually have the knowledge
In order of fully understand the barriers to and expertise necessary to write business plans for
the development of renewable energy enter- projects that link renewable energy technologies
prises/projects in rural India, we must review a with productive use of applications, to assess the
few of the key characteristics of investments in re- rewards and risks of the project and estimate the
newable energy projects. costs to mitigate the associated risks. All of these
are necessary inputs before a project is presented
First, investments in renewable energy projects are to a financier for investment. In addition, the nego-
relatively information and capital-intensive. The tiations with financiers may be complex, involv-
greater information intensity arises primarily from ing concepts and approaches that are not familiar
the need for more upfront information regard- to the entrepreneur. Second, the paperwork and
ing the energy resource compared to conventional soft costs associated with identifying and obtain-
power projects. Renewable energy resource as- ing access to financing for small and medium-scale
sessments, in most cases, need to be site-specific projects is high relative to the financing needs.
and preferably with data for a significant period Without critical first-stage financing, most of the
of time. Renewable energy technologies are also entrepreneurs interested in developing renewable
more capital-intensive than conventional hydrocar- energy projects will not be able to take the time
bon ones because of the large upfront investment away from normal business operations to imple-
cost in generation equipment needed to utilize a ment renewable energy projects. Third, many of
free or low-cost energy source. Due to the cap- the renewable energy technologies are still rela-
ital intensity, the financial viability of such in- tively new to the market, so the commercial chains,
vestments and projects is often more dependent networks, marketing and financial links, and other
on longer-term financing structures available at the institutional structures that service traditional en-
outset of the project. ergy technologies are not in place to assist the en-
Second, individual renewable energy investments trepreneur even if she/he has the skills, know-how
are generally smaller than those made in conven- and capital. Fourth, given the newness, there is a
tional power generation projects due to constraints limited availability of investment capital to finance
on local resource availability. Local resource avail- the high upfront costs associated with the initial
ability is in turn limited by the low energy density stages of developing a renewable energy project.
yoursmahboob.wordpress.com
8 Practice Set 2

71. Which of the following is/are the reason/s that the C. Information needed by these projects is
programmes implemented by IREDA and MNES mostly confidential and requires a lot of paperwork
have not yet succeeded in making RETs the major in order to gain access.
alternative source of energy? D. The information required by these projects is
far greater than that required by any other project
1. Focus on employment generation. related to energy conservation.
2. Emphasis on industrial use of renewable en- E. The information required by these projects is
ergy. very vast and sometimes unrelated to the project
3. Interest rate subsidies linking these to Gov- at hand.
ernment budgets and political cycles.
75. What, according to the passage, is a drawback of
A. Only 2 B. Only 2 and 3 C. Only 3 renewable energy projects newness?
D. Only 1 and 2 E. All 1, 2 and 3 A. The unfamiliarity with the concept of renew-
able energy amongst the locals.
72. Which of the following is possibly the most appro-
B. The low risk factor involved in its implemen-
priate title for the passage?
tation attracts entrepreneurs who fail to complete
A. Programmes to Implement Renewable Energy these projects in time.
Projects in Rural Areas. C. Too many investors willing to invest in these
B. Rural Renewable Energy Projects - The Bar- projects thereby creating confusion.
ries D. The absence of well-established links for
C. The Benefits of Alternative Sources of En- networking, marketing and financing.
ergy E. These require minimum paperwork thus
D. The Government and Renewable Energy there are few or no records of these projects being
Projects generated for future reference.
E. Taking Forward Renewable Energy Projects
in India
Directions(76 78): Choose the word/group of
73. Which of the following is/are the characteristic/s words which is most similar in meaning to the
of investment in RETs? word/group of words printed in bold as used in
the passage.
1. There are a number of investors willing to in-
vest readily invest in these projects. 76. OUTSET
2. These demand more investment of capital A. Start B. Periphery C. Boundary
over the conventional energy projects. D. Culmination E. Stage
3. These involve lesser risk as compared to the
conventional energy projects. 77. VIABILITY
A. Technicality B. Rigidity C. Regularity
A. Only 1 and 3 B. Only 1 and 2 C. Only 2 D. Commodity E. Feasibility
D. Only 2 and 3 E. All 1, 2 and 3
78. RECOGNIZED
74. Why are renewable energy projects termed
information-intensive? A. Predictable B. Standard C. Known
D. Resembled E. Questioned
A. The information on resources required by these
projects is specific in terms of the location as well
as the time period. Directions(79 80): Choose the word/group of
B. Very little information about renewable en- words which is most opposite in meaning to the
ergy is available for the implementation of these word/group of words printed in bold as used in
projects. the passage.
yoursmahboob.wordpress.com
Practice Set 2 9

79. MITIGATE 3. But nowhere were Romas these abilities


A. Aggravate B. Anger C. Moderate recognised on her report cards, because there
D. Alienate E. Alleviate was no check-off box for artwork and creativ-
ity.
80. COMPLEX 4. Some of her artwork were selected to represent
A. Plain B. Simple C. Undisturbed her school at art exhibition at state the level
D. Compound E. Unilateral art exhibition.
Directions(81 85): Read each sentence to 5. As a child, she had been a good student-
find out whether there is any grammatical mis- above average in most areas but not an ex-
take/error in it. The error if any, will be in one ceptional early reader and writer.
part of the sentence. Mark the number of the part
with error as your answer. If there is no error, 86. Which of the following should be the THIRD sen-
mark (E). tence after rearrangement?

81. To run a company effectively (A)/ it is very impor- A. 1 B. 2 C. 3 D. 4 E. 5


tant (B)/ in knowing the strengths and weaknesses
87. Which of the following should be the FOURTH
(C)/ of the employees. (D)/ No error (E)
sentence after rearrangement?
A. A B. B C. C D. D E. E
A. 1 B. 2 C. 3 D. 4 E. 5
82. The land records (A)/ of this district (B)/ will
computerise (C)/ by next year. (D)/ No error (E) 88. Which of the following should be the SECOND
sentence after rearrangement?
A. A B. B C. C D. D E. E
A. 1 B. 2 C. 3 D. 4 E. 5
83. The Head Office has (A)/ issued instructions that
(B)/ the performance of all Zonal Managers (C)/ 89. Which of the following should be the FIRST sen-
have to assess by a committee. (D)/ No error (E) tence after rearrangement?
A. A B. B C. C D. D E. E A. 1 B. 2 C. 3 D. 4 E. 5

84. She has promised to (A)/ donate the funds to (B)/ 90. Which of the following should be the FIFTH sen-
establish a library in many (C)/ villages in India. tence after rearrangement?
(D)/ No error (E) A. 1 B. 2 C. 3 D. 4 E. 5
A. A B. B C. C D. D E. E

85. We have already (A)/ submitted our application Directions(91 95): In each of the following
(B)/ and expect to receive (C)/ our licence in sentence there are two blank spaces. Below each
thirty days.(D)/ No error (E) sentence there are five pair of words denoted by
letters A, B, C, D, and E. Find out which pair of
A. A B. B C. C D. D E. E words can be filled up in the blanks in the sen-
Directions(86 90): Rearrange the following tence in the same sequence to make the sentence
five sentences 1, 2, 3, 4, and 5 in the proper se- meaningfully complete.
quence to form a meaningful paragraph then an-
swer the questions given below them. 91. He objected to the proposal because it was founded
on a .... principle and also was .... at time.
1. What teachers repeatedly commented on was A. faulty .... desirable
that she was very creative" and had an un- B. imperative .... reasonable
usual way of thinking", etc. C. wrong .... inconvenient
2. A study done by the institute focuses on D. sound .... acceptable
Roma, an individual with creative abilities. E. unconforming .... deplorable
yoursmahboob.wordpress.com
10 Practice Set 2

92. The criterion for ..... a player should be his recent (98) in a city like water, electricity, transport etc.
performance, but unfortunately, the journalists are need special consideration.
..... to be carried away by earlier successes. TERI has put together a detailed report that (99)
A. condemning .... satisfying sustainability in the provision of basic urban ser-
B. judging .... prone vices in Indian cities.
C. revealing .... reluctant Inadequate public transport is a major reason for
D. eager .... acclaiming the proliferation of private vehicles on the road.
E. criticising .... clean Respiratory illness in children living in urban areas
93. For the last half century, he ..... himself to public is on the rise with more cases of Asthma being
affairs ..... taking a holiday. (100) because of pollution. The future of cities of
Indian dreams depends on whether we can build
A. by .... committed
better cities today.
B. after .... offered
C. devoted .... without 96. A. Speculated B. Believed C. Imagined
D. sacrified .... after D. Considered E. Understand
E. prepared .... before
97. A. Payment B. Rate C. Costs D. Charge
94. You will see signs of ..... everywhere, which speak E. Expense
well for the .... of these people.
98. A. Abundance B. Large C. Functional
A. decoration .... senses
D. Vicinity E. Location
B. clear .... debris
C. beauty .... careful 99. A. Bring B. Emphasizes C. Speculates
D. industry .... prosperity D. Postulates E. Requests
E. repairs .... extravaganza
100. A. Produced B. Develop C. Composed
95. The police arrested Ramesh on a .... of theft but D. Resulted E. Reported
for lack of evidence .... him.
A. crime .... imprisoned
B. punished .... complaint
C. left .... condition
D. tip .... absconding
E. charge .... released Answers

Directions(96 100): In the following passage 1. B 2. E 3. C 4. B 5. D 6. B 7. D 8. D


there are blanks, each of which has been numbered. 9. B 10. B 11. A 12. B 13. A 14. D 15. D
These numbers are printed below the passage and 16. A 17. C 18. D 19. E 20. A 21. B 22. E
against each five words/phrases are suggested, one 23. A 24. D 25. C 26. A 27. B 28. A 29. A
of which fits the blank appropriately. Find out the 30. D 31. B 32. B 33. C 34. C 35. D 36. C
appropriate word/phrase in each case. 37. A 38. E 39. A 40. C 41. E 42. E 43. B
44. C 45. A 46. C 47. D 48. A 49. E 50. E
Twenty years from now, nearly 60% of the worlds 51. C 52. B 53. C 54. B 55. D 56. C 57. C
population will live in urban areas. The impact 58. B 59. B 60. A 61. D 62. C 63. C 64. C
of urbanization might not all be positive on India 65. D 66. D 67. E 68. E 69. B 70. C 71. C
as urban expansion is happening at a much faster 72. A 73. C 74. D 75. D 76. A 77. E 78. C
rate than infrastructure expansion. 79. E 80. B 81. C 82. E 83. D 84. C 85. D
Sustainability issues need to be (96) so that eco- 86. E 87. A 88. D 89. B 90. C 91. C 92. B
nomic development is not at the (97) of public 93. C 94. D 95. E 96. D 97. E 98. A 99. B
health. Some urban services that ought to be in 100. E
yoursmahboob.wordpress.com
Practice Set 2 11

Detailed Answers 9. (B)


R O A D
Reasoning
+3 +3 +3 +3
1. (B) Brother
G.F
U R D G
S W A N
+3 +3 +3 +3

Son
V Z D Q

Son Boy Veena


10. (B)
F A V O U R
Brother +1 1 +1 1 +1 1
E B U P T S
2. (E) None of these
D A N G E R
Father
+1 1 +1 1 +1 1
daughter
C B M H D S
Sarika
11. (A)
mother daughter
North from initial position
Reeta
7.5 m
Direction(3 5):
north

Create - ba, new - ri, ideas - gi, always - sha,


thoughts - ma, and - fa, insight - jo, better/solution 10 m
- ki/to
starting point
3. (C) ideas - gi
12. (B)
4. (B) Create and innovate
80m 100m
5. (D) insights always better - to sha jo
F 30m
6. (B) One B
60m
E % A 20m
20m
Vowel vowel
D C
7. (D)
x2 = 802 + 602
BCM@N&T2Y6S?Q$7?W#Z3%4 x2 = 6400 + 3600 = 10000

13th place 8th x = 10000
place x = 100 m
8. (D) 25 13. (A)
All characters will become English letters at final. Three words can be formed from AMLE
yoursmahboob.wordpress.com
12 Practice Set 2

1.LAME Step 2: 25 101 85 66 49 73 39 115 74 142


2.MEAL Step 3: 25 39 101 85 66 49 73 115 74 142
3.MALE Step 4: 25 39 101 85 66 49 73 74 115 142
Step 5: 25 39 49 101 85 66 73 74 115 142
14. (D)
Step 6: 25 39 49 85 66 73 74 101 115 142
15. (D) Step 7: 25 39 49 66 85 73 74 101 115 142
16. (A) Step 8: 25 39 49 66 73 74 85 101 115 142
Directions(17 20): 8 steps are required for final output.
B E 25. (C)
Input: 67 23 58 159 46 123 79
D F Step 1: 23 67 58 159 46 123 79
G Step 2: 23 67 58 46 123 79 159
A C Step 3: 23 46 67 58 123 79 159

17. (C) F is 3rd to the right of D is false Directions(26 30):


18. (D) D is to the left of G 1) Alka
2) Gopal
19. (E) None of these
3) Mala
20. (A) 4th to the right of D 4) Ram
21. (B) 5) Kapil
Input: 68 182 39 93 129 46 21 58 26. (A) Malas age is between Gopal and Ram
Step 1: 21 68 182 39 93 129 46 58 27. (B) Rams age is between Mala and Kapil
Step 2: 21 68 39 93 129 46 58 182
28. (A) Malas age is exactly in the middle
Step 3: 21 39 68 93 129 46 58 182
29. (A) Alka is the eldest
22. (E) Cannot be determined.
30. (D) Kapil is the youngest
23. (A)
31. (B) Some dumb asses are bad asses
Input: 138 63 49 93 89 122 32 71
32 138 63 49 93 89 122 71
B S D
32 63 49 93 89 122 71 138
32 49 63 93 89 122 71 138
32 49 63 93 89 71 122 138 32. (B) All lands have houses
32 49 63 71 93 89 122 138
Last step: 32 49 63 71 89 93 122 138

24. (D) C L H

Input: 101 85 66 49 73 39 142 25 115 74


Step 1: 25 101 85 66 49 73 39 142 115 74
yoursmahboob.wordpress.com
Practice Set 2 13

33. (C) All kitchens have shelves 40. (C)


1 [1 + 1 {1 + 1 (1 1)}]
1[1 + 1 2]
2
K C 1 1.5 =
S 3
41. (E)
186 271 50406
= = 1260
40 40
34. (C) Some life savers are not bumper cars
42. (E)
4780
B 529
A L 296
1195
= 529
74
632155
35. (D) None of the conclusions follow = = 8542.63 = 8540
74
43. (B)
D 28
175 28
b 275
P 5488
= 498.9 = 500
11
44. (C)

Numerical Ability 624.99 + 729.95 = 25 + 27 = 52

45. (A)
36. (C)
69 33
1600 1 700 + 400
8500 + = 8501 100 100
x 5 483 + 132 = 615
8500x + 320 = 8501x
x = 320 46. (C)
3
37. (A) 24 = 36
2
377 1 1 1
= 36 2 = 72
13 29 4 2 8 5
72 = 180
38. (E) 2
5 - [5 - {5 5(10)}] 180 3 = 540
7
5 -[5 + 45] 540 = 1890
2
5 - 50 = -45 1890 4 = 7560
39. (A) 47. (D)
175 11 - 50 3 (25) 8 31 122 485 1936 7739 ?(30950)
3
175 11 - 50 8 4 - 1 = 31
25
175 11 - 50 0.12 31 4 - 2 = 122
1925 - 6 = 1919 122 4 - 3 = 485
yoursmahboob.wordpress.com
14 Practice Set 2

485 4 -4 = 1936 52. (B)


1936 4 - 5 = 7739 Cost Price of bike = 4700
7739 4 - 6 = 30950 Repair cost = 800

48. (A) CP = Cost price of bike + Repair cost = 5500

234 448 876 1732 ?(3444) Selling price of the bike = 5800
Gain
234 + 214 = 448 Gain% = 100
CP
448 + (214 2) = 876 Gain = 5800 - 5500 = 300
876 + (428 2) = 1732 300 5
Gain% = 100 = 5 %
5500 11
1732 + (856 2) = 3444
53. (C)
49. (E)
Population increased :
4248 2112 1074 513 286.5 ?(107.25)
262500 - 175000 = 87500
4248 0.5 - 12 = 2112
Average Percent for a decade(1 decade = 10 years)
2112 0.5 + 18 = 1074 87500
= 100 = 50%
1074 0.5 - 24 = 513 175000
513 0.5 + 30 = 286.5 50
Average percent for 1 yr = = 5%
10
286.5 0.5 -36 = 107.25
54. (B)
50. (E)
Let mothers age be M
1140 1148 1175 1239 1364 ?(1580)
Let Sons age be S
1140 + 23 = 1148
M = 3S + 2 M - 3S = 2
1148 + 33 = 1175
M + 3 = 2(S + 3) + 8
1175 + 43 = 1239
M + 3 = 2S + 6 + 8
1239 + 53 = 1364
M = 2S + 11
1364 + 63 = 1580
M - 2S = 11
51. (C) M - 3S = 2
Average income : M - 2S = 11
A+B+C S=9
= 250
3
A + B + C = 750 1 Sub S = 9 in

B earns 30 more than C, M-39=2

B = 30 + C 2 M = 2 + 27

A earns double of C M = 29 yrs

A = 2C - 3 55. (D)
Substitute 2 & 3 in 1 Ratio of 25p, 10p, 5p = 2 : 3 : 4
2C + 30 + C + C = 750 2(0.25)x + 3(0.1)x + 4(0.05)x = 45
4C = 720 0.50x + 0.3x + 0.2x = 45
C = 180 x = 45
yoursmahboob.wordpress.com
Practice Set 2 15

Number of 5p Coins = 4x P(7r + 200) = 232800


= 4 45 232800
P=
= 180 7r + 200
Equating values of P
56. (C) 50400 232800
=
A = 5P r + 50 7r + 200
A=P+I 252 1164
=
r + 50 7r + 200
5P = P + I
21(7r + 200) = 97(r + 50)
I = 4P
147r + 4200 = 97r + 4850
By formula,
50r = 650
100 P
r = 13%
nr SI
PNR
100 P SI =
100
16r 4P P 2 13
1008 - P =
4r = 100 100
100800 - 100P = 26P
r = 25%
126P = 100800
57. (C) 100800
P=
A=P+I 126
= Rs. 800
I=A-P
I = 1008 - P 58. (B)
I = 1164 - P Let x be side of square

(I) n = 2yrs Then its diagonal = x 2 = 1.414x
pnr
SI = By walking through the edges length = 2x
100
P 2r Difference = 2x - 1.414x = 0.586x
1008 - P = 0.586x
100 % Saved = 100
50400 - 50P = Pr 2x
Pr + 50P = 50400 ' 29.5%

P(r + 50) = 50400 59. (B)


50400 A:B:C
P=
r + 50
7 5: 7: 8
(II) n = yrs
2 14x + 8y + 7z = 5 : 7 : 8
I = 1164 - P
14x = 5 8y = 7 7z = 8
P 7r
1164 - P = 5 7 8
2 100 x= y= z=
7pr 14 8 7
1164 - P = 5 7 8
200 x: y: z : :
232800 - 200P = 7Pr 14 8 7
20 49 64
7Pr + 200P = 232800 : : 20 : 49 : 64
56 56 56
yoursmahboob.wordpress.com
16 Practice Set 2

60. (A) 65. (D)


2 12 11 10
30 = 0.6 12C3 = = 220
100 123
0.6 + x 10 5C3 + 4C3 + 3C3 = 10 + 4 + 1 = 15
=
30 + x 100 15 3
60 + 100x = 300 + 10x =
220 44
240 3
x= Probability of same colour =
90 44
2 3 41
= 2 kg Not in same colour = 1 - =
3 44 44
61. (D) 66. (D)
S = 3 km/hr [Village to School] By Statements I, II & III can find salary of Sudha
S = 2 km/hr [School to Village] 15
10000 1500
Let Distance = x km 100
Distance For 5yrs 1500 5 = 7500
Time =
Speed Sudhas present Salary = 10000 + 7500 = 17500
x x
+ =5 67. (E)
3 2
2x + 3x = 30 By using all the statements I, II & III are sufficient
5x = 30 to answer the question
x=6 68. (E)
Distance = 6 km All the statements are necessary to find solution
62. (C) 69. (B)
Divisible by 5 so the number ends with 5 Using Statement II & III alone we can find the cost
4 digit number : - - - 5 of flooring the rectangular hall.
6 5 4 = 120 lb = 336

63. (C) Cost per square metre = 550


1 336 550 = Rs. 1,84,800
A=
80
70. (C)
1 1
As work for 10 days = 10 = 1
80 8 Statement I - A =
1 7 8
Remaining work = 1 - = 1 1 1 1 11
8 8 Statement II - A - B = + =
8 1 5 6 5 6 30
Bs work: B = 30
7 42 =
1 11
B= 1
48 Statement III - B =
1 1 8 16
A+B= + = 1 1 3 16
80 48 240 Statement I & III + = =
= 30 days 8 16 16 3
By using II alone or I and III together, we can find
64. (C) A and B working together days.
U S + DS 8 + 12
Rate in still water = = = 10
2 2
yoursmahboob.wordpress.com

Practice Set 3
Numerical Ability 8. (563% of 808) 129 = ?
A. 63 B. 19 C. 35 D. 47 E. 51
Directions(1 5): What should come in place of
question mark (?) in the following questions? 9. 77.077 7.07 6.08 = ?
A. 57 B. 46 C. 48 D. 77 E. 66
1. 7632 32 + 9248 ? = 306.5
2 2
A. 272 B. 136 C. 102 D. 68 E. None of 10. (16.01) - (8.99) = ?
these A. 175 B. 180 C. 170 D. 165 E. 185
3 2
2. ? = 2268
4 7 Directions(11 15): What should come in place
A. 3136 B. 3481 C. 3249 D. 3364 of the question mark (?) in the following number
E. None of these series?
4.5 0.5 11. 3, 4, 12, ?, 576, 27648
3. =?
0.015 A. 36 B. 48 C. 32 D. 44 E. None of
A. 150 B. 15 C. 0.15 D. 1.5 E. None of these
these
12. 6, 18, 90, 630, ?, 62370
1 11 25
4. 5 +1 =? A. 4900 B. 4800 C. 5400 D. 5600
7 14 45
E. None of these
7 17 23 2
A. 43 B. 6 C. 6 D. 56
9 126 126 7 13. 6, 349, 565, ?, 754, 781
E. None of these
A. 690 B. 601 C. 640 D. 680 E. None
of these
5. 12728 7396 + ? = 269
A. 11 B. 14641 C. 141 D. 1488414. 1.5, 4, 20, 129, ?, 10505
E. None of these A. 1096 B. 1072 C. 1036 D. 1048
Directions(6 10): What approximate value E. None of these
should come in place of the question mark (?) in 15. 89250, 17850, 7140, 1428, ?, 114.24
the following questions? (Note: you are not ex- A. 571.2 B. 489.4 C. 513.9 D. 417.3
pected to calculate the exact value). E. None of these
6. 1164 128 8.008 + 969.007 = ? 16. The average expenditure of a man for the first 12
A. 18800 B. 19000 C. 19600 D. 19200 days of a month was Rs. 850 per day and for the
E. 18600 next 18 days, it was Rs. 825 per day. If he could
save Rs.700 during that month. Find his income
7. 324.995 15.98 4.002 + 36.88 = ? for the month.
A. 1300 B. 1230 C. 1440 D. 1380 A. Rs. 22650 B. Rs. 29450 C. Rs. 24150
E. 1340 D. Rs. 25750 E. None of these

1
yoursmahboob.wordpress.com
2 Practice Set 3

17. The price of a vehicle, passing through three hands period does B join, if the profits at the end of the
rises on the whole by 65%. If the first and the year are divided in the ratio of 3 : 1?
second sellers earned 20% and 25% profit respec- A. 4 months B. 5 months C. 6 months
tively. Find the percentage profit earned by the D. 8 months E. 7 months
third seller.
25. Tea worth Rs.126 per kg and Rs. 135 per kg are
A. 20% B. 5% C. 15% D. 10% E. None
mixed with a third variety in the ratio 1 : 1 : 2. If
of these
the mixture is worth Rs. 153 per kg, the price of
18. If the numerator of fraction is increase by 20% and the third variety per kg will be:
denominator is decrease by 80% then fraction be- A. Rs. 169.50 B. Rs. 170 C. Rs. 175.50
comes 18/15. What is the actual fraction? D. Rs. 140 E. Rs. 174.50
A. 2/3 B. 1/5 C. 4/3 D. 8/5 26. A motor car does a journey in 10 hrs, the first half
E. None of these at 21 kmph and the second half at 24 kmph. Find
19. Thirteen years ago, a father was three times as old the distance.
as his son. Now the father is only twice as old as A. 442 km B. 224 km C. 108 km D. 216
his son. Then the sum of the present ages of the km E. None of these
son and the father is:
27. 48 workers can reap a field in 9 days. If the work
A. 72 years B. 64 years C. 78 years D. 84 is to be completed in 6 days, the extra workers
years E. None of these required are:
20. The sum of the three members daily salary is 267. A. 36 B. 32 C. 24 D. 22 E. None of
If the ratio of the first person to second person is these
4 : 5 and that of the second person to the third 28. A man, a woman and a boy can together complete
person is 6 : 7 the second persons monthly salary a piece of work in 5 days. If a man alone can do
will be: it in 12 days and a boy alone in 15 days, how long
A. 2070 B. 900 C. 2670 D. 2700 will a woman take to complete the work?
E. 2580 A. 20 days B. 24 days C. 30 days D. 34
days E. None of these
21. The difference between the compound interest and
simple interest on a certain sum at 10% per annum 29. A man rows 50 km downstream and 40 km up-
for 2 years is Rs. 631. Find the sum. stream taking 10 h each time. What is the rate of
current?
A. Rs. 52255 B. Rs. 50000 C. Rs. 63200
D. Rs. 63100 E. None of these A. 0.5 km/hr B. 1 km/hr C. 1.5 km/hr
D. 2 km/hr E. None of these
22. The rectangular mat has an area of 120 sq.m. and
perimeter of 46 m. The length of its diagonal is: 30. A box contains 4 red balls, 5 green balls and 6
white balls. A ball is drawn at random from the
A. 17 cm B. 17 m C. 17.5 m D. 16.5 m box. What is the probability that the ball drawn
E. Cannot be determined is either red or green?
23. The ratio of the areas of two squares, one having A. 2/5 B. 3/5 C. 1/5 D. 7/5 E. None
double its diagonal than the other is: of these
A. 1 : 3 B. 3 : 1 C. 1 : 4 D. 4 : 1
E. None of these Directions(31 35): Each of these questions
consists of a question followed by information in
24. A began a business with Rs. 85000. He was joined three statements. You have to study the question
after wards by B with Rs. 42500. For how much and the statements and decide that information in
yoursmahboob.wordpress.com
Practice Set 3 3

which of the statements is/are necessary to answer III. Selling price per article was Rs. 765.
the questions. A. II and III B. I and II C. Question cant
be answered even with the information all three
31. What is the capacity of the cylindrical tank?
statements
I. Radius of the base is half of its height. D. Any two of the three E. All of the above
II. Area of the base is 616 sq.m.
III. Height of the cylinder is 28 m. Reasoning Ability
A. I and II B. II and III C. I and III
D. Any two of the three E. All of these 36. If FRIEND is coded as HUMJTK, how can CAN-
DLE be written in that code?
32. What is the speed of the train?
A. DEQJQM B. DCQHQK C. EDRIRL
I. The train crosses a signal pole in 18 s. D. ESJFME E. None of these
II. The train crosses a platform of equal length in
37. If in a certain code, LUTE is written as MUTE and
36 s.
FATE is written as GATE, then how will BLUE be
III. Length of the train is 300 m. written in that code?
A. I and III B. II and III C. I and II A. CLUE B. GLUE C. FLUE D. SLUE
D. Either I and III or I and II E. Any two of E. ALUE
the three
38. A told B, The girl I met yesterday was the
33. What is the staff strength of Company X? youngest daughter of the brother-in-law of my
friends mother. How is the girl related to As
I. Male and female employees are in the ratio of 2
friend?
: 3 respectively.
A. Niece B. Cousin C. Friend D. Daugh-
II. Of the officer employees 80% are males. ter E. None of these
III. Total number of officers is 132.
A. I and III B. II and either III or I C. Ques- 39. A man is facing west. He turns 45 degree in the
tion cant be answered even with the information clock wise direction and then another 180 degree
all three statements in the same direction and then 270 degree in the
D. Any two of the three E. All of the above anti clock wise direction. Find which direction he
is facing now?
34. What is the two digits number?
A. South west B. West C. South D. East-
I. Number obtained by interchanging the digits is
South E. None of these
more than the original number by 9.
Directions(40 44): Study the following infor-
II. Sum of the digits is 7. mation carefully and answer the given questions.
III. Difference between the digits is 1. A word and number arrangement machine when
given an input line of words and numbers rear-
A. I and III B. I and II C. II and III
ranges them following a particular rule in each
D. Question cant be answered even with the in-
step. The following is an illustration of input and
formation all three statements
rearrangement.
E. All of the above
Input : say dry 42 96 get 39 kite 67
35. How many articles were sold? Step I : 96 say dry 42 get 39 kite 67
Step II : 96 dry say 42 get 39 kite 67
I. Total profit earned was Rs. 1596. Step III : 96 dry 39 say 42 get kite 67
II. Cost price per article was Rs. 632. Step IV : 96 dry 39 say 67 42 get kite
yoursmahboob.wordpress.com
4 Practice Set 3

Step V : 96 dry 39 say 67 get 42 kite 45. Which of the following is false?
Step V is the last step of the rearrangement. A. A is fourth to the right of E
You have to answer the questions by following the B. G is to the immediate right of D
same rules as illustrated above. C. F is the third to the right of D
D. B is the immediate left of D
40. If the second step of an input is 76 from 48 super
E. None of these
itself 56 18 went", how many more steps will be
required to complete the arrangement? 46. Which of the following is true?
A. Five B. Six C. Four D. Three A. C is fourth to left of B
E. None of these B. A is to the immediate right of G
C. D is second to the left of E
41. Which step will be the last step of an input for
D. B is second to the right of G
which the third step is "91 go 28 mock pet 43 lead
E. None of these
37"?
A. Eighth B. Seventh C. Sixth D. Fifth 47. Which of the following pair has the first person
E. None of these sitting to the immediate left of the second person?
A. BE B. CA C. GD D. DC E. None
42. What will be the third step if the input is "thirty
of these
days from now 32 56 87 24"?
A. 87 thirty days from now 32 56 24 B. 87 days 48. Which of the following is the positions of F?
thirty from now 32 56 24 C. 87 days 24 thirty A. Fourth to the right of D
from now 32 56 D. 87 thirty 24 days 32 from B. To the immediate left of C
now 56 E. None of these C. Between A and C
D. To the immediate right of A
43. If the third step of an input is "65 daily 12 tie 42 23 E. None of these
foreign urgent", what will definitely be the input?
Directions(49 51): Each of the questions be-
A. foreign 65 tie urgent 12 42 23 daily B. foreign low consists of a question and two statements num-
65 urgent tie 42 daily 23 12 C. foreign 65 12 bered I and II are given below it. You have to de-
urgent tie 42 daily 23 D. Cannot be determined cide whether the data provided in the statements
E. None of these are sufficient to answer the question.
44. If the second step of an input is "52 at deep follow Read Both statements and
41 16 road 32", what will be the fifth step?
1. Give answer A) If the data in statement I
A. 52 at 16 road 32 deep follow 41 B. 52 at
alone are sufficient to answer the question,
16 road 41 deep follow 32 C. 52 at 16 road 32
while the data in statement II alone are not
follow 41 deep D. There will be no such step
sufficient to answer the question.
E. None of these
2. Give answer B) If the data in statement II
Directions(45 48): Study the following infor- alone are sufficient to answer the question,
mation carefully to answer the given question. while the data in statement I alone are not
sufficient to answer the question.
1. A, B, C, D, E, F and G are sitting around a
circle and are facing the centre. 3. Give answer C) If the data either in statement
I alone or in statement II alone are sufficient
2. G is second to the left of C, who is to the to answer the question.
immediate left of F.
4. Give answer D) If the data in both the state-
3. A is third to the left of E. ments I and II are not sufficient to answer the
4. B is between D and E questions.
yoursmahboob.wordpress.com
Practice Set 3 5

5. Give answer E) If the data in both the state- c) If either I or II is true


ments I and II together are necessary to an-
d) If neither I or II is true
swer the question.
e) If both I and II is true
49. Who among P, Q, R, S, T, V and W is the shortest?
52. Statements: M @ R, R % T, T $ K
1. S is taller than T, P and W and is not the
Conclusion:
tallest.
2. T is shorter than Q but is not the shortest. I. KM
II. TM
A. A B. B C. C D. D E. E
A. A B. B C. C D. D E. E
50. What is Mrudulas rank from top in a class of
twenty students? 53. Statements: H % J, B + J, B @ F
Conclusion:
1. Radhika is fifth from the top and two ranks
above Mrudula I. F $ J
2. Amol is tenth from the bottom and three II. J % F
ranks below Mrudula.
A. A B. B C. C D. D E. E
A. A B. B C. C D. D E. E
54. Statements: D $ M, M % W, W @ R
51. Who among M, N, O P and Q is the youngest? Conclusion:
1. N, the second youngest, is younger than Q, O I. R D
and M.
II. W + D
2. O, the second oldest, is older than N.
A. A B. B C. C D. D E. E
A. A B. B C. C D. D E. E
55. Statements: A + N, N V, V $ J
Directions(52 56): In these questions symbols
@, +, %, and $ are used with different meanings Conclusion:
as follows: I. J @ N
P @ Q means P is either greater than or equal to II. A + V
Q
A. A B. B C. C D. D E. E
p + Q means P is either smaller than or equal to
Q 56. Statements: K T, T @ B, B + M
P % Q means P is greater than Q
Conclusion:
P Q means P is smaller than Q
I. M % T
P $ Q means P is neither greater than nor smaller
than Q II. K + B

Now in each of the following questions assuming A. A B. B C. C D. D E. E


the given statement to be true, find which of the Directions(57 60): Study the following ar-
two conditions I and II given below them is/are rangement carefully and answer the questions
definitely true? Give answer. given below:
a) If only conclusion I is true RE5DAP$3TIQ79B#2K%U1MW
b) If only conclusion II is true 4*J8N
yoursmahboob.wordpress.com
6 Practice Set 3

57. How many such numbers are there in the above ar- All the instruments are flutes
rangement, each of which is immediately preceded Conclusions:
by a consonant and not immediately followed by a
consonant? 1. All the flutes are instruments
A. None B. One C. Two D. Three 2. All the harmoniums are flutes
E. None of these
A. Only (1) conclusion follows B. Only (2) con-
58. How many such vowels are there in the above ar-
clusion follows C. Either (1) or (2) follows
rangement, each of which is immediately followed
D. Neither (1) nor (2) follows E. Both (1) and
by a number but not immediately precede by a
(2) follow
consonant?
A. None B. One C. Two D. Three 63. Statements:
E. Four Some ants are parrots.
59. Which of the following is seventh to the left of the All the parrots are apples.
sixteenth from the left in the above arrangement? Conclusions:
A. A B. U C. 4 D. T E. None of these
1. All the apples are parrots
60. Four of the following five are alike in a certain way
2. Some ants are apples
based on their position in the above arrangement
and so form a group. Which is the one that does
A. Only (1) conclusion follows B. Only (2) con-
not belong to that group?
clusion follows C. Either (1) or (2) follows
A. M * U B. D P E C. W J 1 D. 3 Q P D. Neither (1) nor (2) follows E. Both (1) and
E. None of these (2) follow
Direction(61 65): In each of the question be-
64. Statements:
low are given two statements followed by three con-
clusions numbered I and II. Read all the conclu- Some papers are pens.
sions and then decide which of the given conclu- All the pencils are pens.
sions logically follows from the given statements
disregarding commonly known facts. Conclusions:

61. Statements: 1. Some pens are pencils.


Some actors are singers. 2. Some pens are papers.
All the singers are dancers
A. Only (1) conclusion follows B. Only (2) con-
Conclusions: clusion follows C. Either (1) or (2) follows
D. Neither (1) nor (2) follows E. Both (1) and
1. Some actors are dancers (2) follow
2. No singer is actor
65. Statements:
A. Only (1) conclusion follows B. Only (2) con- All the actors are girls.
clusion follows C. Either (1) or (2) follows All the girls are beautiful.
D. Neither (1) nor (2) follows E. Both (1) and
(2) follow Conclusions:

62. Statements: 1. All the actors are beautiful.


All the harmoniums are instruments. 2. Some girls are actors.
yoursmahboob.wordpress.com
Practice Set 3 7

A. Only (1) conclusion follows B. Only (2) Certain words have been printed in bold to help
conclusion follows C. Either (1) or (2) follows you locate them while answering some of the ques-
D. Neither (1) nor (2) follows E. Both (1) and tions.
(2) follow Once upon a time, there was a stork that lived
by catching fish from a particular tank. As the
Directions(66 70): years went by, the stork grew older and weaker and
B, M, T, R, K, H and D are travelling in a Train its ability to catch fish diminished. At times it
compartment with III-tier sleeper berth. Each of would even starve. The stork knew that it had to
them as a different profession of engineer, doc- do something to survive.
tor, architect, pharmacist, lawyer, journalist and One day it stood by the side of the tank with a
pathologist. They occupied two lower berth, three very forlorn look on its face. The frogs, fish and
middle berth and two upper berth. B, the engi- crabs wondered why it was not trying to catch any
neer, is not the upper berth. The architect is the food. A big crab asked the stork what the matter
only other person who occupies the same type of was. The stork answered that it was sad because
berth as that of B. M and H are not on the mid- all the fish in the tank were going to die and so it
dle berth and their professions are pathologist and would have to starve. The stork said that it had
lawyer respectively. T is a pharmacist. D is nei- heard that people were going to fill the tank with
ther a journalist nor an architect. K occupies the mud and grow crops over it. The fish were very
same type of berth as that of doctor. worried and asked the stork to help them.
66. Who is Architect? The stork offered to take all of them to a bigger
tank some distance away. But it said that it needed
A. D B. H C. R D. Data inadequate
to rest between trips because of old age. It would
E. None of these
only be able to carry a few fish at a time.
67. What is Ds profession? On its first trip, the stork took a beakful of fish. It
A. Pharmacist B. Lawyer C. Doctor flew to a big rock and had a good meal. Thereafter,
D. Engineer E. None of these it rested for a while, and when it was hungry again,
the stork took a second trip. In this manner, the
68. Which of the following pairs occupy the lower
stork took a trip each time it was hungry.
berth?
The big crab in the tank also wanted to save itself
A. BT B. BD C. BK D. Data inadequate
and it requested the stork to take it too. The stork
E. None of these
thought it was a good idea to try a new dish. It
69. Which of the following groups occupies the middle agreed to take the crab on its next trip.
berth? After the stork flew up with the crab, the crab
A. DKT B. HKT C. DKR D. DHT looked down to see what the new surroundings
E. None of these would be like. All it could see was dry land. When
the crab questioned the stork about this, the stork
70. Which of the following combinations of person-
laughed wickedly and pointed to the rock below
berth-profession is correct?
where the crab sawa heap of fish bones. The crab
A. K-upper-lawyer B. D-upper-doctor realized that it was to be the storks next meal.
C. M-lower-journalist D. R-lower-architect So the crab dug its claws into the storks neck and
E. None of these would not let go till the stork fell to the ground and
died. The crab then cut off the storks head and
General English returned home to show it to all the other fish and
share the story of its adventure with the stork.
Directions(71 78): Read the following passage Directions(71 72): Choose the word which is
carefully and answer the questions given below it. most similar in meaning to the word printed in
yoursmahboob.wordpress.com
8 Practice Set 3

bold as used in the passage. aquariums.


D. They would fill the tank with mud and
71. ADVENTURE cultivate crops.
A. Incident B. Decision C. Saviour E. None of these
D. Scheme E. Friendship

72. CATCHING Directions(79 88): In the following passage


there are blanks, each of which has been numbered.
A. Teasing B. Holding C. Detecting
These numbers are printed below the passage and
D. Sipping E. Eating
against each, five words are suggested, one of which
fits the blank appropriately without changing its
Directions(73 74): Choose the word which is meaning. Find out the appropriate word in each
most opposite in meaning to the word printed case.
in bold as used in the passage.
Many kinds of insects are ...(79)... and larger
73. DIMINISHED animals learn to ...(80)... eating them. It is
in the habit of these insects to show a ...(81)...
A. Praised B. Developed C. Agitated
flag of some kind. The typical wasp, for example,
D. Excited E. Lessened
...(82)... a conspicuous colour pattern of black
74. FORLORN and yellow bands on its body. This is ...(83)...
distinctive that it is easy for a ...(84)... animal to
A. Serious B. Satisfied C. Wicked
remember it. After a few unfortunate experiences
D. Helpful E. Cheerful
it quickly learns to avoid insects bearing this pat-
75. Why did the stork agree to take the crab along? tern. Other, unrelated, poisonous insect species
A. It wanted to try a new dish may alos carry a similar pattern. They become
B. It wanted to prove that it was still young members of what has been called a warning club.
C. It wanted to save the crab The important point for us, in the present
D. It was blackmailed by the crab ...(85)... is that some harmless species of insects
E. None of these have taken advantage of this system by developing
colour patterns similar to those of the poisonous
76. Which of the following can be the most appropriate members of the warming club. Certain innocuous
title for the story? files, for instance, ...(86)... black and yellow bands
A. The foolish fish B. The brave stork on their bodies that mimic the colour patterns of
C. The crabs adventure with the stork D. The the wasps. By becoming ...(87)... members of the
innocent stork and the fish E. The lazy crab warning club they reap the benefits without hav-
ing possess any real poison. The killers dare not
77. Why did the stork have to remain hungry? attack them, even though they would, in reality,
A. There were not fish in the tank. make a ...(88)... meal.
B. The storks capacity to catch fish had dimin-
ished. 79. A. flying B. conspicuous C. vulnerable
C. The stork had lost its teeth due to old age. D. big E. poisonous
D. The fish had learnt to trick the stork.
80. A. desist B. adjust C. prefer D. avoid
E. None of these
E. nurture
78. As per the stork what had the people being saying
81. A. dangerous B. ambiguous C. warning
about the tank?
D. coloured E. safety
A. The tank would dry due to immense heat.
B. The tank would be poisoned by the people. 82. A. reveal B. paints C. flashes D. carried
C. They would trap the fish and take them to E. loaded
yoursmahboob.wordpress.com
Practice Set 3 9

83. A. so B. extremely C. normally D. too Directions(94 100): Pick out the most effec-
E. very tive pair of words from the given pair of words to
make the sentence/s meaningfully complete.
84. A. bold B. prey C. chased D. predatory
E. hunted 94. Participative management, in which everyone has
..... into a decision that a leader then makes, is a
85. A. text B. premise C. context D. day mechanism for .... employees.
E. view
A. share, protecting B. value, thewarting
86. A. display B. indicates C. announce C. motivation, involving D. reward, stimu-
D. weal E. bears lating
E. input, empowering
87. A. unregistered B. fake C. honorary
D. sycophant E. original 95. Lack of ...... is basic to good teamwork but our
ability to work with others depends on our.....
88. A. troublesome B. delicious C. indigestible
A. rigidity, compatibility
D. hazardous E. cheap
B. dogmatism, motivation
C. professionalism, vulnerability
Directions(89 93): Read each sentence to D. positivism, flexibility
find out whether there is any grammatical mis- E. consideration, acumen
take/error in it. The error if any, will be in one
part of the sentence. Mark the number of the part 96. Complete and constant openness is a nation that
with error as your answer. If there is no error, can be ..... to absurdity. Am I ..... to stop everyone
mark (5). on the street and tell them my reaction to their
appearance?
89. The cost of constructing (1)/ houses are increased
A. consigned, communicated B. reduced, re-
(2)/ because of the high (3)/ price of cement. (4)/
quired
No error (5)
C. attributed, requested D. projected, des-
A. 1 B. 2 C. 3 D. 4 E. 5 tined
E. subjected, confined
90. According to the Twelfth (1)/ Five year plan, In-
dia should (2)/ invest one trillion dollars (3)/ in 97. When organizations ..... creativity and risk taking,
infrastructure projects. (4)/ No error (5) the usual method of maintaining order and ..... are
A. 1 B. 2 C. 3 D. 4 E. 5 indeed shaken.
A. encourage, decorum B. exhibit, durability
91. To increase the selling (1)/ of products in rural ar-
C. propose, humility D. enhance, supply
eas (2)/ the company will hire (3)/ over five hun-
E. propagate, production
dred trainees. (4)/ No error (5)
A. 1 B. 2 C. 3 D. 4 E. 5 98. When I am an autocrat, I am ..... in the extreme.
My direct method is to ..... power and control.
92. We have spent (1)/ most of the profits (2)/ that we A. perfectionist, explore B. autocratic, engulf
earn (3)/ last year on purchasing new computers. C. dominating, seek D. possessive, reject
(4)/ No error (5) E. elaborate, develop
A. 1 B. 2 C. 3 D. 4 E. 5
99. Gopal was frustrated with Sundar who would not
93. The Government has (1)/ promised to revise (2)/ ..... himself to a deadline. Sundar claimed he was
the pension scheme for bank (3)/ staff since next working well without a deadline, but Gopal .... and
year. (4)/ No error (5) finally prevailed
A. 1 B. 2 C. 3 D. 4 E. 5 A. encourage, enforced B. inculcate, ordered
yoursmahboob.wordpress.com
10 Practice Set 3

C. cooperate, stipulated D. commit,persisted Detailed Answers


E. declare, pressurized
Numerical Ability
100. I am not easily ..... by pressure that would in-
terfere with accomplishing the goals of my unit. I 1. (B)
stick with my .....
7632 32 + 9248 ? = 306.5
A. pessimistic, views
238.5 + 9248 x = 306.5
B. swayed, convictions
C. discouraged, achievements 9248
= 68
D. empowered, organization x
E. demurred, projections x = 136

2. (E)
3 2
x = 2268
4 7
x = 10584

3. (A)
4.5 0.5
=?
0.015
2250
= 150
15
4. (B)
36 25 25
+ =?
7 14 45
773 17
=6
126 126
5. (A)

12728 7396 + x = 269
Answers 12728
+ x = 269
86
1. B 2. E 3. A 4. B 5. A 6. C 7. E 8. C
x = 121 , x = 11
9. E 10. A 11. B 12. E 13. A 14. D 15. A
16. D 17. D 18. B 19. C 20. D 21. D 22. B 6. (C)
23. D 24. D 25. C 26. B 27. C 28. A 29. A
1164 128 8.008 + 969.007 = ?
30. B 31. B 32. D 33. C 34. B 35. E 36. C
37. A 38. B 39. A 40. C 41. B 42. C 43. D 18624 + 969 = 19593
44. B 45. C 46. B 47. E 48. A 49. D 50. C Approximate = 19,600
51. A 52. E 53. C 54. A 55. D 56. D 57. C
58. B 59. D 60. B 61. A 62. B 63. B 64. E 7. (E)
65. E 66. C 67. C 68. E 69. A 70. D 71. A
324.995 15.98 4.002 + 36.88
72. E 73. B 74. E 75. A 76. B 77. B 78. D
164
79. E 80. D 81. C 82. C 83. A 84. D 85. C 325 + 37
86. D 87. B 88. B 89. B 90. D 91. A 92. C 4
= 1300 + 37 = 1337
93. D 94. A 95. A 96. B 97. D 98. C 99. D
100. B approximate = 1340
yoursmahboob.wordpress.com
Practice Set 3 11

8. (C) 4 4 + (2)2 = 16 + 4 = 20
(563% of 808) 129 = x 20 6 + (3)2 = 120 + 9 = 129
x = 35 129 8 + (4)2 = 1032 + 16 = 1048

9. (E) 15. (A)


77.077 7.07 6.08 = ? 89250, 17850, 7140, 1428, x, 114.24
11 6 = 66 89250 5 = 17850
17850 2.5 = 7140
10. (A)
7140 5 = 1428
(16.01)2 - (8.99)2 = ?
1428 2.5 = 571.2
256 - 81 = 175
16. (D)
11. (B)
Income spent for first 12 days = 12 850 = 10200
3, 4, 12, ?, 576, 27648
Income spent for the next 18 days = 18 825 =
3, 4, 12, x, 576, 27648 14850
3 4 = 12 Total income = 10200 + 14850 + 700 (savings)
4 12 = 48 = Rs. 25750
12 48 = 576
17. (D)
12. (E) Let the price of the vehicle be Rs. 100
6 18 90 630 5670 62370 Then the whole rise in the prize = 165
the first and second person earns 120 and (120
6 3 18 5 90 7 630 9 5670 11 125
) 150 respectively
6, 18, 90, 630, x, 62370 100
6 3 = 18 The 3rd person gains 165 - 150 = Rs. 15
x
18 5 = 90 150 = 15
100
90 7 = 630 x = 10%
630 9 = 5670 18. (B)
13. (A) 120
x x
6 349 565 x 754 781 becomes 100
y 20
y
100
+7 3 +6 3 +5 3 +4 3 +3 3 120
x 18
6 + 343 (73 ) = 349 100 =
20 15
y
349 + 216 (63 ) = 565 100
x
565 + 125 (53 ) = 690 = 1/5
y
14. (D)
19. (C)
1.5, 4, 20, 129, ?, 10505 13 years ago, fathers age be x and Sons age will
1.5, 4, 20, 129, x, 10505 be y
1.5 2 + (1)2 = 3 + 1 = 4 x - 13 = 3(y - 13)
yoursmahboob.wordpress.com
12 Practice Set 3

x = 3y - 26 l = 8; l = 15
Now, x = 2y b = 15; b = 8
p
3y - 26 = 2y diagonal = (15)2 + (8)2 = 225 + 64

3y - 2y = 26 = 289 = 17m
y = 26 23. (D)
x = 52 d2
Area of the square having its diagonal as d =
Sum of the present ages = 26 + 52 2
= 78 years double the length of diagonal means = 2d
4d2
20. (D) Area =
2
Sum of daily salary of three members = Ratio = 4 : 1
(A + B + C) = 267
24. (D)
Ratio of First person to second person
85000 12 : 42500 x = 3 : 1
A:B=4: 5
2 12 : 1 x = 3 : 1
Ratio of second person to third person 24 3
=
B:C=6: 7 x 1
x = 8 months
4: 5
6: 7 25. (C)
126 + 135
24 : 30 : 35 Mixer of 1st and 2nd Variety is = 130.50
2
Second persons daily salary 130.5 + x
30 Third Variety mixer is = 153
= 267 = 90 2
89 x = 175.5
Monthly salary = 90 30 = 2700
26. (B)
21. (D) distance
Speed =
pr2 time
= 631
1002 distance
p 10 10 Time =
= 631 speed
100 100 x/2 x/2
p = 63100 10 = +
21 24
x x
22. (B) 10 = +
42 48
120 7x + 8x
Rectangular area = l b = 120 sq.m = b = 10 =
l 336
Perimeter = 2(l + b) = 46 m 15x
120 10 = x = 224
2l + 2( ) = 46 336
l
27. (C)
2l2 + 240 = 46l
1
l2 + 120 23l = 0 48 workers 1 days work =
9
l2 15l 8l + 20 = 0 48 9 = (48 + x) 6
l(l 15) 8(l 15) = 0 72 = 48 + x
yoursmahboob.wordpress.com
Practice Set 3 13

x = 24 34. (B)
24 extra workers to be added. I and II are sufficient to answer the question
28. (A) 35. (E)
1 All I, II and III are required to answer the question
M+W+B=
5
1
1 Mans 1 day work =
12 Reasoning
1
1 boys 1 day work =
15 36. (C)
1 1 1
+ +x= F R I E N D
12 15 5
9 1 1 9
+x= x= - +2 +3 +4 +5 +6 +7
60 5 5 60
3 1 H U M J T K
x= =
60 20 C A N D L E
Woman can complete the work in 20 days.
+2 +3 +4 +5 +6 +7
29. (A)
50 E D R I R L
= 10
a+u
1 1 37. (A)
=
a+u 5 L U T E
40 M U T E
= 10
au
1 1 F A T E
= G A T E
au 4
a+u =51 B L U E
a-u=42 C L U E
By solving 1 & 2, a = 4.5 and u = 0.5 km/hr
38. (B) Cousin
30. (B)
39. (A) South west
Probability that the ball will be Red or Green
4C1 or5C1 Direction(40 44)
=
15C1 40. (C)
9 3
= = II- 76 from 48 super itself 56 18 went
15 5
III- 76 from 18 48 super itself 56 went
31. (B) IV- 76 from 18 went 48 super itself 56
II and III are sufficient to answer the question V- 76 from 18 went 56 48 super itself
32. (D) VI- 76 from 18 went 56 itself 48 super
Either I and III or I and II 41. (B)
33. (C) III- 91 go 28 mock pet 43 lead 37
Question cant be answered IV- 91 go 28 pet mock 43 lead 37
even with the information all three statements V- 91 go 28 pet 43 mock lead 37
yoursmahboob.wordpress.com
14 Practice Set 3

VI- 91 go 28 pet 43 lead mock 37 50. (C)


VII- 91 go 28 pet 43 lead 37 mock From statement I

42. (C) Mrudula rank = 7th (5+2)


Input- Thirty days from now 32 56 87 24 From Statement II
I- 87 Thirty days from now 32 56 24 Mrudula rank = 7th
II- 87 days Thirty from now 32 56 24 Either statement I alone or II alone are sufficient
to answer the question
III-87 days 24 Thirty from now 32 56

43. (D) 51. (A)

Cannot be determined Statement I:


May be M > O > Q > N > P
44. (B)
Statement II:
III- 52 at 16 deep follow 41 road 32
>O>N>>
IV- 52 at 16 road deep follow 41 32
So there is no clue for youngest.
V- 52 at 16 road 41 deep follow 32
Direction(45 48): Statement I alone is sufficient to answer the ques-
tion
E F
Direction(52 56):
+ $
B . C < =
> =
% @ $

D A 52. (E)
G M@R%T$K

45. (C) F is third to the right of D I) K x M - follow


II) T x M - follow
46. (B) A is to the immediate right of G
Both I & II is true
47. (E) None of these
53. (C)
48. (A) F is fourth to the right of D
H%J@B@F
Direction(49 51):
I) F $ J - does not follow
49. (D) II) J % F - does not follow
From the statement I Either I or II is true
S > T, P and W
54. (A)
Statement II
D$M%W@R
Q>T
I) R x D - follow
So,the shortest may be T or P or W or V
Statement I & II are not sufficient to answer the II) W + D - does not follow
question Only conclusion I is true
yoursmahboob.wordpress.com
Practice Set 3 15

55. (D) I - does not follow


A+NxV$J II - follows
I) J @ N - does not follow Conclusion II follows
II) A + V - does not follow 63. (B)
Neither I nor II is true

56. (D) A P App


nt
KxT@B+M
I) M % T - does not follow
I - does not follow
II) K + B - does not follow
II - follows
Neither I nor II is true
Conclusion II follows
Direction(57 60): 64. (E)
57. (C) Two Pen
Q79
Paper Pencil
W4

58. (B) One


I - follows
%U1
II - follows
59. (D) T Both I & II follow
60. (B) 65. (E)
DPE
In all other groups, the first element moves 3 steps beautiful
forward to give the second elements which in turn g
moves 2 steps backward to give the third element. a
Direction(61 65):

61. (A)
I - follows
A S d II - follows
Both I & II follow
I - follows Direction(66 70):
II - does not follow B Engineer lower berth
Conclusion I follows M Pathologist upper
62. (B) H Lawyer upper
T Pharmacist middle
F
D Doctor middle
H I K Journalist middle
R Architect lower
yoursmahboob.wordpress.com
16 Practice Set 3

Upper Berth M H
Middle Berth D K T
Lower Berth R B

66. (C) R - Architect

67. (C) D - Doctor

68. (E) Lowerberth - B,R

69. (A) Middleberth - T D K

70. (D) R - lowerberth - architect


yoursmahboob.wordpress.com

Time : 1 hrs. Max. Marks : 100

QUANTITATIVE APTITUDE (a) 4.485 (b) 2.589 (c) 4 (d) 2


(e) None of these
DIRECTIONS (Qs. 1-10) : What will come in place of the question
mark (?) in the following equations ? (272 - 32)(124 + 176)
6. =?
17 15 15
(a) 0 (b) 2.25 (c) 300 (d) 240
117 117 117 - 98 98 98
1. =? (e) None of these
117 117 + 117 98 + 98 98
(a) 215 (b) 311 (c) 19 (d) 29 50 ?
7. =
(e) None of these ? 1
12
2
a 4 3a + 2b
2. If = , then =?
b 3 3a - 2b 25 4
(a) (b) (c) 4 (d) 25
(a) 6 (b) 3 (c) 5 (d) 1 2 25
(e) None of these (e) None of these

? 72 112 576 256


3. = 8. =?
196 56 196 12 8

(a) 18 (b) 14 (c) 324 (d) 212 (a) 8 (b) 12 (c) 16 (d) 32
(e) None of these (e) None of these

17.28 ? ?
4. = 200 9. = 550
3.6 0.2 2.25
(a) 120 (b) 1.20 (c) 12 (d) 0.12 (a) 825 (b) 82.5 (c) 3666.66 (d) 2
(e) None of these (e) None of these

(3.537 - 0.948)2 + (3.537 + 0.948)2 5- 3


=? 10. =?
5. 5+ 3
(3.537)2 + (0.948)2
(a) 4 + 15 (b) 4 - 15
yoursmahboob.wordpress.com
1
(c) 6500 (d) 4800
(c) (d) 1 (e) None of these
2
15. The population of village C is 2000 in 1995. What will be the
(e) None of these ratio of population of village C below poverty line to that of
DIRECTIONS (Qs. 11-15) : Study the following chart to answer the village E below poverty line in that year ?
the questions given below : (a) 207 : 76 (b) 76 : 207
(c) 152 : 207 (d) Data inadequate
(e) None of these
Villages % population below poverty line
16. A train is moving at a speed of 132 km/h. If the length of the
A 45 train is 110 metres, how long will it take to cross a railway
B 52 platform, 165 metres long ?
C 38
D 58 (a) 5s (b) 7.5 s (c) 10 s (d) 15 s
E 46
(e) None of these
F 49
17. If 15 women or 10 men can complete a project in 55 days, in
G 51
how many days will 5 women and 4 men working together
complete the same project ?
Proportion of population of seven villages in 1995
(a) 75 (b) 8 (c) 9 (d) 85
G A (e) None of these
15% 13% 18. Ashus mother was three times as old as Ashu, 5 years ago.
After 5 years, she will be twice as old as Ashu. How old is
Ashu at present?
F B
(a) 15 (b) 20 (c) 10 (d) 5
13% 16%
(e) None of these
19. A conical flask has base radius a cm and height h cm. It
is completely filled with milk. The milk is poured into a
C
cylindrical thermos flask whose base radius is p cm. What
E 8%
will be the height of the solution level in the flask ?
18% D
a 2h 3hp2 p2 3a 2
17% (a) cm (b) cm (c) cm (d) cm
3p 2
a 2
3h 2 hp 2
11. In 1996, the population of villages A as well as B is increased (e) None of these
by 10% from the year 1995. If the population of village A in 20. A sum was put at simple interest at a certain rate for 2 years.
1995 was 5000 and the percentage of population below Had it been put at 3% higher rate, it would have fetched
poverty line in 1996 remains same as in 1995, find ` 300 more. Find the sum.
approximately the population of village B below poverty (a) ` 6000 (b) ` 8230 (c) ` 5000 (d) ` 4600
line in 1996. (e) None of these
(a) 4000 (b) 45000
DIRECTIONS (Qs. 21-25) : Identify which number is wrong in
(c) 2500 (d) 3500
the given series.
(e) None of these
12. If in 1997 the population of village D is increased by 10% 21. 2, 3, 4, 4, 6, 8, 9, 12, 16.
and the population of village G is reduced by 5% from 1995 (a) 3 (b) 9
and the population of village G in 1995 was 9000, what is the (c) 6 (d) 12
total population of villages D and G in 1997? (e) None of these
(a) 19770 (b) 19200
22. 3, 4, 10, 32, 136, 685, 41
(c) 18770 (d) 19870
(a) 136 (b) 10
(e) None of these
(c) 4116 (d) 32
13. If in 1995 the total population of the seven villages together
was 55,000 approximately, what will be population of village (e) None of these
F in that year below poverty line ? 23. 69, 55, 26, 13, 5
(a) 3000 (b) 2500 (a) 26 (b) 13
(c) 4000 (d) 3500 (c) 5 (d) 55
(e) None of these (e) None of these
14. If the population of village C below poverty line in 1995 was 24. 24576, 6144, 1536, 386, 96, 4
1520, what was the population of village F in 1995 ? (a) 386 (b) 6144
(a) 4000 (b) 6000 (c) 96 (d) 1536
yoursmahboob.wordpress.com
(e) None of these 33. Anish spends 25% of his salary on house rent, 5% on food,
25. 11, 5, 20, 12, 40, 26, 74, 54 15% on travel, 10% on clothes and the remaining amount of
(a) 5 (b) 20 ` 22,500 is saved. What is Anish's salary ?
(c) 40 (d) 26 (a) ` 40,000 (b) ` 40,500
(e) None of these (c) ` 45,500 (d) ` 50,000
(e) None of these
DIRECTIONS (Qs. 26-30): Find out the approximate value which
should come in place of the question mark in the following 2
34. th of Anil's salary is equal to Bhuvan's salary and seven-
questions. (You are not expected to find the exact value. ) 5
ninth of Bhuvan's salary is equal to Chandra's salary. If the
26. 45689 = ? sum of the salary of all of them is ` 77,000, then, how much
(a) 180 (b) 415 is Bhuvan's salary?
(c) 150 (d) 210 (a) ` 45,000 (b) ` 18,000
(e) 300 (c) ` 15,000 (d) ` 28,000
(e) None of these
(10008.99)2 35. A tap can fill an empty tank in 12 hours and a leakage can
27. 3589 0.4987 = ?
10009.001 empty the whole tank in 20 hours. If the tap and the leakage
are working simultaneously, how long will it take to fill the
(a) 3000 (b) 300000 whole tank?
(c) 3000000 (d) 5000 (a) 25 hours (b) 40 hours
(e) 9000000 (c) 30 hours (d) 35 hours
28. 399.9+ 206 11.009= ? (e) None of these
(a) 2800 (b) 6666 REASONING ABILITY
(c) 4666 (d) 2400
(e) 2670
2 7 17 6
29. + =?
5 8 19 5

1
(a) 1 (b)
2

1 3
(c) 2 (d)
2 4

9
(e)
11
30. (299.99999)3 = ?
(a) 27000000 (b) 9000000000
(c) 180000 (d) 2.7 109
(e) 2700000
31. A reduction of 20% in the price of sugar enables a purchaser
1
to obtain 2 kg more for ` 160. Find the original price per
2
kg of sugar.

(a) ` 12 (b) ` 20 (c) ` 16 (d) ` 18


(e) None of these
32. Mrs. X spends ` 535 in purchasing some shirts and ties for
her husband. If shirts cost ` 43 each and the ties cost ` 21
each, then what is the ratio of the shirts to the ties, that are
purchased ?
(a) 1 : 2 (b) 2 : 1 (c) 2 : 3 (d) 3 : 4
(e) None of these
yoursmahboob.wordpress.com
(e) None of these II. Some buds are flowers.
41. Ankit is related to Binny and Chinky, Daizy is Chinkys III. Some leaves are trees.
mother. Also Daizy is Binnys sister and Aruna is Binnys (a) Only II and III follow
sister. How is Chinky related to Aruna? (b) Only III follows
(a) Niece (b) Sister (c) Only either I or II follows
(c) Cousin (d) Aunt (d) Either I or II and III follow
(e) None of these (e) None of these
42. Rama remembers that she met her brother on Saturday, which 48. Statements: All stones are rocks.
was after the 20th day of a particular month. If the 1st day of Some rocks are bricks.
that month was Tuesday, then on which date did Rama Some bricks are cement.
meet her brother ? Conclusions: I. Some cements are rocks.
(a) 24th (b) 23rd II. Some bricks are stone
(c) 25th (d) 26th III. Some stones are cement.
(e) None of these (a) Only I and either II or III follow
43. If it is possible to make only one such number with the first, (b) Only either II or III follows
the fourth and the sixth digits of the number 531697 which (c) Only I and II follow
is the perfect square of a two digit even number, which of (d) All follow
the following will be the second digit of the two digit even (e) None of these
number. If no such number can be made, give '@' as the 49. Statements: All flats are buildings.
answer and if more than one such number can be made, All buildings are bungalows.
give '' as the answer. All bungalows are apartments.
(a) 4 (b) 2 Conclusions: I. Some apartments are flats.
(c) 6 (d) @ II. All flats are bungalows.
(e) III. Some bungalows are flats.
44. In a certain code JOURNEY is written as TNISZFO. How is (a) None follows
MEDICAL written in that code? (b) Only I and II follow
(a) CDLJMBD (b) CDWDBM (c) Only II and III follow
(c) LDCJMBD (d) EFNJMBD (d) Only I and III follow
(e) None of these (e) All I, II and III follow
45. If 'K' denotes '', 'B' denotes '', 'T' denotes '' and 'M' 50. Statements: Some spectacles are lenses.
denotes '+', then
Some lenses are frames.
40 B 8 T 6 M 3 K 4 = ?
All frames are metals.
(a) 19 (b) 11
Conclusions: I. Some lenses are metals
(c) 31 (d) 23
II. Some metals are spectacles.
(e) None of these
III. Some frames are spectacles.
DIRECTIONS (Qs. 46-50): In each question below are three (a) Only III follows
statements followed by three conclusions numbered I, II and III. (b) Only I follows
You have to take the three given statements to be true even if (c) Only I and either II or III follow
they seem to be at variance from commonly known facts and then (d) Only I and II follow
decide which of the answers (a), (b), (c), (d) and (e) is the correct (e) None of these
answer and indicate it on the answer sheet.
46. Statements: Some chairs are tables. DIRECTIONS (Qs. 51 - 55) : Read the following information
Some tables are drawers. carefully and answer the questions that follow:
all drawers are shelf.
Conclusions: I. Some shelves are tables. At a party, A, B, C, D and E are sitting in a circle. The group
II. Some drawers are chairs. comprises a professor, an industrialist and a businessman. The
III. Some shelves are drawers. businessman is sitting in between the industrialist and his wife
(a) Only I and III follow D. A, the professor is married to E, who is the sister of B. The
(b) Only I and either II or III follow industrialist is seated to the right of C. Both the ladies are
(c) Only II and either I or III follow unemployed.
(d) All I, II and III follow 51. What is A to B ?
(e) None of these (a) Brother (b) Uncle
47. Statements: All trees are flowers. (c) Brother-in-law (d) Cant be determined
Some flowers are leaves. (e) None of these
No leaf is bud 52. A is sitting to the right of
Conclusions: I. No bud is a flower.
yoursmahboob.wordpress.com
(a) the industrialist (b) his wife and ) has a value equivalent to the square of the numeral
(c) D (d) Cant be determined that immediately precedes the symbol or the value of 1 if it
(e) None of these is not immediately preceded by a numeral, what will be the
53. Who is the industrialist ? sum of the values of the first 10 elements of the series starting
(a) D (b) A from the left end ?
(c) B (d) Cant be determined (a) 118 (b) 46
(e) None of these (c) 79 (d) 107
(e) None of these
54. Who in the group is unmarried?
63. If each of the letters in the above series of elements is given
(a) Professor (b) Industrialist
a value equivalent to its serial number in the English
(c) Businessman (d) Cant be determined
alphabet, what will be the difference between the sum of
(e) None of these
the consonants and the sum of the vowels used in the
55. Who among them must be graduate ?
series ?
(a) B (b) A
(a) 109 (b) 41
(c) C (d) E
(c) 82 (d) 27
(e) None of these
(e) None of these
DIRECTIONS (Qs. 56-60): In the questions given below, certain 64. Which of the following groups of elements will come in the
symbols are used with the following meanings: place of the question-mark in the series of elements given
A @ B means A is greater than B. below?
A * B means A is either greater than or equal to B.
682 $B5 E Q9 ? D7
A # B means A is equal to B.
A $ B means A is either smaller than or equal to B. (a) (13) 4 (b) 4 (13)
A + B means A is smaller then B. (c) U D (d) (13)
Now in each of the following questions, assuming the (e) None of these
given statements to be true, find which of the two 65. BQ in the above series is related in ER in a similar way as AP
conclusions I and II given below them is/are definitely true? is related to
(a) If only conclusion I is true (a) RD (b) U
(b) If only conclusion II is true (c) KA (d) Q 3
(c) If either conclusion I or II is true (e) 6 B
(d) If neither conclusion I nor II is true
(e) If both conclusions I and II are true DIRECTIONS (Qs. 66-70) : Study the following information
56. Statements : B + D; E$T; T * P; P@B carefully to answer these questions.
Conclusions : I. P$D A group of people has six family members and an advocate.
II. P@D These are L, M, N, O, P, Q and R and having different professions.
57. Statements : E*F; G$H; H#E; G@K Each one of them is a journalist, businessman, architect, doctor
Conclusions : I. H @K and pilot but not necessarily in this order. There are three males
II. H*F and three females in the family out of which there are two married
58. Statements : P$Q; N#M; M@R; R*P
couples. M is a businessman and is the father of P. N is a housewife
Conclusions : I. P + N
and is daughter-in-law of O. L is neither a pilot nor a journalist. R
II. Q$M
is an advocate. N is not the mother of P and O is not married to M.
59. Statements : D + T; E $V; F *T; E@D
Conclusions : I. D $ V No lady is a journalist.
II. D + F 66. Which of the following groups represents the three ladies
60. Statements : T*U; U$W; V @L; W + V in the group ?
Conclusions : I. V @ T (a) N, P, L (b) P, L, N
II. L #W (c) L, N, O (d) O, P, L
DIRECTIONS (Qs. 61-65): Use the following series of elements (e) None of these
(alpha-number-symbol) to answer these questions. Every two- 67. Who is married to Q ?
digit number (given in brackets) is to be treated as single number.
(a) N (b) O
2 8 5 6 B 9 $ Q 3 E 1 7 R D 4 (13) U K (18) A (14) P (c) L (d) Cant be determined
61. Four of the following five groups of elements are alike in a (e) None of these
certain way and so form a group. Which is the one the does
not belong to that group ? 68. Who among the following family members is an architect ?
(a) 2 8 (b) 5 6 B (a) L (b) O
(c) Q $ 9 (d) 1 3 E (c) P (d) Cant be determined
(e) D R 7 (e) None of these
62. If each alphabet has a value of zero, each symbol (i.e., , $ 69. Which of the following is the profession of P ?
yoursmahboob.wordpress.com
(a) Architect (b) Pilot would be able to sell the ornaments for a good price. But who could
(c) Architect or pilot (d) Journalist help him to sell the ornaments? He then remembered Seth
(e) None of these Ghanshyamdas. He went to him. The goldsmith was glad to see
Krishnan. I have come to ask for your help, said Krishnan. Here
70. How is Q related to O ?
are some ornaments. Please give me a good price for them. Seth
(a) Father (b) Mother
Ghanshyamdas took the jewellery and examined it carefully. I shall
(c) Mother-in- law (d) Son - in - law certainly help you, he said. But let me show them to another
(e) None of these goldsmith. Please wait here, I will be right back. He then went out
ENGLISH LANGUAGE with the ornaments. Seth at once rushed to the Palace of the King.
He said, A man brought these ornaments to me and asked me to
DIRECTIONS (Qs. 71-80): Read the following passage carefully sell them. But they are the ornaments I made for the Prince who is
and answer the questions given below it. missing. Who is this man? Where is he?, thundered the King.
This rogue must have murdered my little Prince and robbed his
Long ago there was a poor Brahmin named Krishnan. He
jewels! He is a Brahmin named Krishnan, your Majesty, replied
could not find enough work to do. Sometimes, he and his family
the goldsmith, and he is there, in my house. The king called for his
had to go without food. At last Krishnan decided to leave his
most dreaded soldiers. Arrest the Brahmin who is in the
village in search of work. Early next morning, he left the house.
goldsmiths house and throw him into the darkest dungeons of the
He walked the whole day until he came to a thick jungle. He was kingdom, roared the King. The Kings guard stormed into the
tired, thirsty and hungry. While looking around for water to goldsmiths house and seized Krishnan. Krishnan was thrown into
drink, he found a well. He went to the well and looked in. There he a dark dungeon to await his execution. He then remembered the
saw a jaguar, a monkey, a snake and a man. They had all fallen words of Naagesh, the snake. So he called out to him.
into the well. O, noble Brahmin, the jaguar called out to him,
Suddenly, almost like magic, Naagesh slithered his way
Please help me out, so that I can go back to my family.
down a narrow window into the dingy cell. O, Lord! hissed
But you are a jaguar, said Krishnan. How do I know you Naagesh, how did you manage to get yourself arrested?
will not kill me? Dont be afraid of me, I promise I will not do Krishnan cried and then told the snake what had happened. I
you any harm, replied the jaguar. Krishnan reached into the well have a plan, hissed Naagesh. I shall creep into the Queens
and pulled out the jaguar. The jaguar thanked him and said, Im room and bite her, said Naagesh. She will faint. No matter what
Shersingh. I live in a cave in the mountains. I shall be most they do, she will remain asleep. The poison will remain in her
delighted if I can repay my debt to you someday. Krishnan then body until you place your hand on her forehead, explained
heard the monkey calling out to him from the well. The Brahmin at Naagesh. He then left Krishnan and went to the palace. He crept
once pulled the monkey out. The monkey thanked the Brahmin. into the Queens room and bit her. The Queen fainted. The sad
If you are ever in need of food, just drop in at my place below news that the Queen had been bitten by a snake spread all over
that big mountain. Bali is my name. Now the snake called out to the Kingdom. Vaidyas came from far and near, but their medicines
him for help. Help you! exclaimed Krishnan. You are a snake. had no effect. No one could revive the Queen. Finally, the King
What if you bite me? I shall never bite you, said the snake. So declared that anyone who could cure the Queen would be
Krishnan pulled the snake out of the well. The snake said, handsomely rewarded. Many people went to the palace but all of
Remember, if you are ever in any difficulty, just call out my them failed. I can cure the Queen, Krishnan told the guards. At
name-Naagesh, and wherever you are, I shall find you. The jaguar, once they took him to the Queen. Krishnan sat beside the Queen
the monkey and the snake took leave of the Brahmin. But before and placed his hand on her forehead. Soon, she opened her eyes
they left, they spoke to him about the man in the well. Please do and sat up. The King was overjoyed and shed tears of happiness.
not help him, said Shersingh. If you do, said Naagesh, you He embraced Krishnan and thanked him. Your Majesty, said
will be in trouble yourself. As soon as they left, the man in the Krishnan. I was sent to prison for a crime I did not commit.
well began to call out for help. Krishnan felt sorry for the man and Krishnan told the King the whole story. The King was fuming
pulled him out of the well. Thank you for your kindness, said with rage when he heard what the goldsmith had done. He at
the man. I am Seth Ghanshyamdas. I am a goldsmith. If you ever once had the goldsmith arrested. The King then presented
need my help, dont hesitate to visit my humble house near the Krishnan with a large house and a thousand pieces of gold.
city. The goldsmith then left for home. Krishnan sent for his family and they all lived happily ever after.
After some time, the Brahmin continued his journey. But he 71. Why did Krishnan decide to leave his village?
could not find any work. He then remembered Shersingh, Bali, (a) As he could not find much work in his own willage and
Naagesh and Seth Ghanshyamdas. He thought it was time to his family had to starve sometimes because of it.
seek their help. He first went to Bali. The monkey was overjoyed (b) As his family had requested him to do so.
to see him. He gave him a warm welcome and offered him some
(c) As his village people had asked him to leave their village
really delicious fruits. The Brahmin told him how grateful he was.
and look for work somewhere else.
Now Krishnan went to see Shersingh, the jaguar. As soon as
Shersingh saw Krishnan coming, he ran out to welcome him. He (d) As he wanted to search for food in a village different
from his own.
gave Krishnan a beautiful gold necklace and other precious
jewellery. Krishnan thanked Shersingh for the jewellery and (e) None of the above
departed. His journey had at last brought him luck, he thought. He 72. Why did the jaguar, the monkey and the snake tell Krishnan
yoursmahboob.wordpress.com
not to save the man in the well? (d) He congratulated the snake on his efforts to save
(a) As the man in the well was a goldsmith Krishnan
(b) As the man in the well had cheated the snake, the (e) None of the above
monkey and the jaguar 79. What did the King do to save the Queen after even the
(c) As the man in the well was a thief Vaidyas failed to revive her?
(d) As the snake, the monkey and the jaguar hated the man (a) He punished the snake for having harmed the Queen
as they had known him for a very long time (b) He announced a reward to anyone who could cure the
(e) None of the above Queen
73. Why was krishnan afraid to save Naagesh from the well? (c) He immediately called for Krishnan to cure the Queen
(a) As Naagesh had thr eatened h im with dire (d) He asked his guards to immediately look for someone
consequences. who could cure the Queen
(b) As he thought Naagesh would eat him. (e) None of the above
80. What can possibly be the moral of the story?
(c) As he thought Naagesh would bite him once he was
out of the well. (a) Trust oneself before trusting overs
(d) As he thought that Naagesh would capture him as soon (b) A good deed never goes in vain
as he got out of the well. (c) You cannot change people but you can change yourself
(e) None of the above. (d) Try and try until you succed
74. Why did Krishnan go to meet Seth Ghanshyamdas? (e) One must be the change one wishes to see in this world
(a) As he thought that Seth Ghanshyamdas could help DIRECTIONS (Qs. 81-85): In each question below, a sentence
him in selling the ornaments gifted to him by Shersingh. with four words printed in bold type is given. These are numbered
(b) As he knew that Seth Ghanshyamdas had contact with as (a), (b), (c) and (d). One of these four words printed in bold may
the King which could prove to be beneficial. be either wrongly spelt or inappropriate in the context of the
(c) As Seth Ghanshyamdas had requested krishnan to sell sentence. Find out the word which is wrongly spelt or
ornaments only to him inappropriate, if any. The number of that word is your answer. If
(d) As Krishnan was extremely fond of Seth Ghanshyamdas
all the words printed in bold are correctly spelt and also
(e) None of the above.
appropriate in the context of the sentence, mark (e) ie. All
75. What did Bali do after seeing Krishnan at his house?
correct as your answer.
(1) He gave Krishnan directions to Shersinghs house.
(2) He welcomed Krishnan to his house. 81. The whole (a)/ time she walked with her child in her arms,
(3) He offered tasty fruits to Krishnan. the only thing (b)/ that worried (c)/ her was her sons
(a) Only 1 (b) Only 2 feature. (d)/ All correct (e)
(c) Only 3 (d) Only 2 and 3 82. When the young artist returned (a)/ to his village, his
(e) 1 and 3 family held a festive (b)/ dinner on its lawn to celebrate his
76. What plan did Naagesh have to save Krishnan from the triumpant (c)/ homecoming. (d)/ All correct (e)
dungeon? 83. Had she not suppressed (a)/ all the details of her Companys
(a) That he would sneak Krishnan out of the dungeon project (b)/ her Company would have bagged (c)/ the
without anyone noticing contract. (d)/ All correct (e)
(b) That he would bite the King and make him unconscious
84. She trusted Mira with all her heart (a)/ and thus handled
(c) That he would bite Krishnan and make everyone believe
(b)/ over her lifes (c)/ savings to her instantly. (d)/ All
that he was dead
(d) That he would enter the Queens chamber and scare correct (e).
her 85. It is difficullt (a)/ to see the picture (b)/ when you are inside
(e) None of the above (c)/ the frame. (d)/ All correct (e)
77. What did Seth Ghanshyamdas tell the King about Krishnan?
(a) That Krishnan had brought fake ornaments for selling DIRECTIONS (Qs. 86-95): In the following passage there are
(b) That krishnan was an honest Brahmin who had left his blanks, each of which has been numbered. these numbers are
village printed below the passage and against each, five words are
(c) That Krishnan had killed the Prince suggested, one of which fits the blank appropriately. Find out
(d) That Krishnan had brought those ornaments for selling the appropriate word in each case.
which had been made for the missing Prince One day a father of a very wealthy family (86) his son on a
(e) None of the above
trip to the country with the purpose of (87) his son how the poor
78. What did the King do on learning the truth about Krishnan
people live so he could be thankful for his wealth. They spent a
and Seth Ghanshyamdas?
(88) of days and nights on the farm of what would be considered
(a) He put Krishnan back in the dungeon as he still held
a (89) poor family. On their (90) from the trip, the father asked his
Krishnan responsible for the Princes death
(b) He called for Krishnans wife and family son, How was the trip?" "It was great, Dad. Did you see how
(c) He presented gold to Krishnan and also a house to live in poor people can be?, the father asked. Oh yeah, said the son.
So what did you (91) from the trip?, asked the father. The son
yoursmahboob.wordpress.com
answered, I saw that we have one dog and they had four. We 95. (a) minds (b) selves
have a pool that (92) in the middle of our garden and they have a (c) property (d) pillars
creek that has no end. We have imported lanterns in our garden (e) country
and they have the stars at night. Our patio reaches to the front
yard and they have the (93) horizon. we have a small piece of DIRECTIONS (Qs. 96 - 100): In each of the following sentences,
land to live on and they have fields that go beyound our sight. an idiomatic expression or a proverb is highlighted. Select the
We have (94) who serve us, but they serve others. We buy alternative which best describes its use in the sentence.
our food, but they grow theirs. We have walls around our (95) 96. He resigned the post of his own accord.
to protect us; they have friends to protect them. (a) which he liked
With this the boys father was speechless. Then his son (b) according to his convenience
added, Thanks dad for showing me how poor we are. (c) voluntarily and willingly
86. (a) took (b) beat (d) according to his judgement
(c) drag (d) mould (e) None of these
(e) showed 97. As a politician he is used to being in the limelight all the time.
87. (a) presenting (b) requesting (a) giving speeches
(c) tell (d) trusting (b) the object of admiration
(e) showing (c) the centre of attraction
88. (a) two (b) couple (d) an object of public notice
(c) much (d) few (e) None of these
(e) many 98. I ran out of money on my European tour.
89. (a) major (b) some (a) exhausted my stock of (b) did not have enough
(c) sorrow (d) very (c) lost (d) carried a lot
(e) astutely (e) None of these
90. (a) lane (b) journey 99. Madhuri might scream blue murder, but I feel Deepali
(c) leave (d) return should get the promotion since she is better qualified for
(e) walking the job.
91. (a) reveal (b) think (a) someone has been murdered with some blue liquid
(c) saw (d) believe (b) someone is being murdered and has become blue
(e) learn (c) suffer from persecution complex
92. (a) stands (b) reaches (d) make a great deal of noise and object vehemently
(c) swims (d) leak (e) None of these
(e) watery 100. In modern democratic societies lynch law seems to have
become the spheres of life.
93. (a) more (b) scene
(a) law of the mob (b) law of the underworld
(c) whole (d) last
(c) law of the constitution(d) law of the parliament
(e) lucky
(e) None of these
94. (a) servants (b) mother
(c) computers (d) relatives
(e) man
yoursmahboob.wordpress.com
Answer Key
1 (c) 11 (d) 21 (b) 31 (c) 41 (a) 51 (c) 61 (d) 71 (a) 81 (d) 91 (e)
2 (b) 12 (a) 22 (d) 32 (b) 42 (d) 52 (d) 62 (a) 72 (e) 82 (c) 92 (a)
3 (c) 13 (d) 23 (c) 33 (d) 43 (a) 53 (c) 63 (b) 73 (c) 83 (a) 93 (c)
4 (d) 14 (c) 24 (a) 34 (b) 44 (a) 54 (d) 64 (e) 74 (a) 84 (b) 94 (a)
5 (d) 15 (b) 25 (c) 35 (c) 45 (b) 55 (b) 65 (c) 75 (d) 85 (a) 95 (c)
6 (c) 16 (b) 26 (d) 36 (b) 46 (a) 56 (c) 66 (c) 76 (e) 86 (a) 96 (c)
7 (d) 17 (a) 27 (b) 37 (e) 47 (c) 57 (e) 67 (b) 77 (d) 87 (e) 97 (c)
8 (d) 18 (a) 28 (e) 38 (b) 48 (e) 58 (a) 68 (a) 78 (c) 88 (b) 98 (a)
9 (a) 19 (a) 29 (a) 39 (b) 49 (e) 59 (b) 69 (d) 79 (b) 89 (d) 99 (d)
10 (b) 20 (c) 30 (a) 40 (b) 50 (b) 60 (d) 70 (e) 80 (b) 90 (d) 100 (a)

HINTS & SOLUTIONS


( a3 - b3 ) x x
, 9. (a) Let = 550. Then, = 550
1. (c) Given Expression = 2.25 1.5
(a 2 + ab + b 2 )
where a = 117, b = 98 550 15
\ x = (550 1.5) = = 825
10
(a - b)(a 2 + ab + b 2 )
= = (a - b) = (117 98) = 19.
(a 2 + ab + b 2 ) ( 5 - 3) ( 5 - 3) ( 5 - 3) ( 5 - 3)2
10. (b) = =
2. (b) Dividing numerator as well as denominator by b, we get: ( 5 + 3) ( 5 + 3) ( 5 - 3) (5 - 3)
a 4
3 + 2 3 + 2 5 + 3 - 2 15 2(4 - 15)
3a + 2b b 3 4+2 = = = (4 - 15)
= = = =3 2 2
3a - 2b 3 a - 2 3 4 - 2 4 - 2
b 3 16
11. (d) Population of village B in 1995 = 5000 6150
13
x 72 9
3. (c) Let = = .
196 56 7 110
Population of village B in 1996 = 6150 = 6750
100
x 9 9 81 81 196
Then , = = . So, x = = 324. Population below poverty line = 52% of 6750 3500
196 7 7 49 49
17
17.28 x 17.28 12. (a) Population of village D in 1995 = 9,000 = 10, 200
4. (d) Let = 200 . Then, = 200 3.6 0.2 15
3.6 0.2 x
17.28 1728 110
\ x= = = 0.12 Population of village D in 1997 = 10, 200
200 3.6 0.2 200 36 2 100
= 11, 220
(a - b)2 + (a + b)2 2(a2 + b2 )
5. (d) Given Expression = = =2 95
(a2 + b2 ) (a2 + b2 ) Population of village G in 1997 = 9,000 = 8,550
100
240 300 \ Total population of village D and G in 1997
6. (c) Given Expression = = 300
240 = 11,220 + 8,550 = 19, 770
25 13. (d) Population of village F below poverty line
50 x 2
7. (d) Let = or x = 50 = 625.
x 25 2 13 49
= 55000 3500
2 100 100
14. (c) Population of village F in 1995
\ x = 625 = 25.
112 24 16 100 13
= 32 = 1520 = 6500
8. (d) Given Expression = 38 8
14 12 8
yoursmahboob.wordpress.com
15. (b) Population of village C below poverty line 3 4 10 32 136 685 4116
38 22. (d)
= 2000 = 760 1+1 2+2 3+3 4+4 5+5 66
100
Thus, 32 is out of place and must be replaced by 33.
Population of village E below poverty line
5
2000 46
= 18 = 2070 23. (c)
8 100
6 9 + 1 5 5 + 1 2 6 +1 1 3 + 1
Thus, 5 does not fit in the series and should be
760
\ Required ratio = = 76 : 207 replaced by 4.
2070 24. (a) The succeeding numbers are obtained by dividing the
preceding numbers by 4. Therefore, the number 386
132 5
16. (b) Speed of the train = 132 km/h = m/s does not fit in the series and must be replaced by 384.
18 25. (c) There are two series in the given series :
Distance = (110 + 165) = 275 m I 5 12 26 54
Time required to cross the railway platform
275 18 2+2 2+2 2+2
= = 7.5 s
132 5 II 11 20 40 74
17. (a) 15 W = 10 M
22 22 22
4 15 Hence the wrong term is 40.
Now, 5W + 4M = 5W + W = 5W + 6W
10 26. (d) ? = 45689 = 213.75 210
= 11 W
Now, 15 women can complete the project in 55 days, (10008.99)2
27. (b) ? = 3589 0.4987
then 11 women can complete the same project in 10009.001
55 15 = (10009) 2 3600 = 0.50
= 75 days
11 = 10009 60 0.50 300000
18. (a) Let the present ages of Ashus mother and that of 28. (e) ? = 399.9 + 206 11.009
Ashu be x and y, respectively. = 400 + (200 + 6) 11 = 400 + 2200 + 66 = 2670
Then, (x5) = 3(y 5) or x 5 = 3y 15 2 7 17 6 2 7 17 5
or x 3y = 10 ...(i) 29. (a) ?= + = +
5 8 19 5 5 8 19 6
and (x + 5) = 2 (y + 5)
And x + 5 = 2y + 10 or x 2y = 5 ...(ii) 2 595
= + = 0.40 + 0.65 1.05 1
From (i) and (ii), we have x = 35 and y = 15 5 912
Hence, the present age of Ashu = 15 years 30. (a) ? = (299.99999)3 (300)3 = 27000000
19. (a) Volume of the conical flask = Volume of the cylindrical 31. (c) Total amount used for purchasing = ` 160. A reduction
flask upto the required height (x) cm of 20% in the price means, now a person gets 5/2 kg
for ` 32 and this is the present price of the sugar.
1 2 2
pa h = pp 2 x x = ha cm 32
3 3p 2 \ Present price per kg = 2 = ` 12.8
5
20. (c) Let the sum = Rs. x and original rate = y % per annum Let the original price be ` x. Then new price is arrived
then, New rate = (y + 3)% per annum after reduction of 20% on it.
x ( y + 3) 2 x y 2 x 0.8 = 12.8 or x = ` 16.
\ - = 300 32. (b) Mrs. X spends = ` 535
100 100
\ Total cost = 43 shirt + 21 ties = 535
xy + 3x - xy = 15000 By hit and trial, S = 10, T = 5
Total cost = 43 10 + 21 5 = 535
\ x = 5000 Thus, the sum = ` 5000 Hence, Ratio of shirts to ties = 10 : 5 = 2 : 1
33. (d) Total expense percentage = (25 + 5 + 15 + 10)% = 55%
2 2
Savings % = 100 55 = 45%
21. (b) 2 3 4 4 6 8 8 12 16 Q 45 22500
2 2 22500
2 2 \ 100% 100 = ` 50000
45
yoursmahboob.wordpress.com
34. (b) Let Anil's salary be ` x. 44. (a) J O U R NE Y
2x 1 +1 +1
\ Bhuvan's salary = `=
5 T N I S Z F O
2 x 7 14 x Similarly,
Chandra's salary = `= =
5 9 45 ME D I C A L
2 x 14 x 1 +1 +1
\ Anil : Bhuvan : Chandra = x : : = 45 :18 :14
5 45 C D L J MB D
\ Bhuvan's salary 45. (b) 40 B 8 T 6 M 3 K 4 = ?
18 ? = 40 + 8 6 + 3 4
= `= 45 + 18 + 14 77000 = ` 18000
( ) ? = 5 + 6 12 = 11
35. (c) Part of the tank filled in an hour 46. (a)
Tables
1 1 53 1 Chairs ers
= = = aw
Dr
12 20 60 30
Hence, the tank will be filled in 30 hours
Hence conclusions I. II. III.
36. (b) There are 25 numbers in the given sequence.
So, middle number = 13th number = 8.
Trees
Clearly, the third number to the left of this 8 is 2. Leaves
47. (c) rs Buds
37. (e) Coding for: I D E A S we
Flo
1 +1 1 +1 1
Hence conclusions I. II. III.
H E D B R
But I and II are complementary pairs.
Coding for: W O U L D
1 +1 1 +1 1
Stone
V P T M C
48. (e) Cement
s Bricks
Similarly, R I G H T ck
Ro
1 +1 1 +1 1 Hence conclusions I. II. III.
Q J F I S
38. (b) Cancelling every second letter after reversing the 49. (e)
alphabet the series becomes.
ZXVT RPNLJHFDB
The middle letter is N.
39. (b) Total no. of girls = 17 + 10 1 or 18 + 9 1 = 26. Flats
40. (b) From first 2 sentences Ka Ya means very intelligent.
From 1st and 3rd sentences Pumeans you Buildings
\ In first sentence are means Bi
Bunglows

41. (a) Daizy Binny Aruna Apartments


(Binnys sister (Binnys sister)
and Chinkys Mother) Hence conclusions I. II. III.

50. (b)

Chinky
Spectacles
42. (d) 1st of month was Tuesday, hence the date on first Frames
Saturday was 5th.
Hence the other Saturdays of the month are 12, 19,
26. Rama met her brother on 26th. es
Lenc
als
Met
43. (a) 5 3 1 6 9 7
576 = 24 24 Hence conclusions I. II. III.
yoursmahboob.wordpress.com
For (Qs. 51 to 55) Middle element and last element also follow the same
trend. Hence (?) should be replaced by R 13.
C 65. (c) Number of elements between B and Q is the same as
(B Man) that between E and R. Similarly, number of elements
between A and P is the same as that between K and A.
Wif For (Qs. 66 - 70) : The given information can be tabulated as
B e D
(Ind) follows
Sis
ter
Wife Pers on S ex Relations hip Profes sion
(Pro) A E L Female W ife of M A rchitect

A, the professor is married to E and E is the sister of B. Father of P. Business man


The wife of the industrialist is D. The industrialist is seated M M ale Husband of L.
to the right of C. Thus, A and C cannot be industrialists.
Son of Q and O.
Therefore, B is the industrialist and C is the business man.
Now, we come to the following deductions : Female Daughter-in-law of
N Hous ewife
A Professor B Industrialist O and Q.
C Businessman D Female, hence unemployed O Female W ife of Q Pilot
E Female, hence unemployed P M ale Son of M and L. Journalis t
51. (c) A is the husband of E and E is the sister of B. Hence,
A is the brother-in-law of B. Grandfather of P. Doctor
52. (d) It cannot be determined, as no information has been Q M ale Husband of Q.
provided in the paragraph about the sitting position
Father of M .
of A.
53. (c) As deduced earlier, B is the industrialist. R A dvocate
54. (d) It cannot be determined whether the businessman is
married or unmarried. Two married couples : LM and QO.
55. (b) As A is a professor, he must be a graduate. 71. (a) Refer to the sentence He could not find.........in search
56. (c) B < D ...(i), E T ...(ii), T P ....(iii), P > B...(iv) From of work.
(i) and (iv), we get, P > B < D no conclusion. But 72. (e) Refer to the sentence Please do not help him..........of
the second para of the passage.
the exhaustive possibilities are P > D , P = D and 73. (c) Refer to the sentence You are...........bite me?........of
P < D. Hence either I or II is true. the second para of the passage.
57. (e) E F ...(i), G H ...(ii), H = E ...(iii), G > K ...(iv) 74. (a) Refer to the sentence He would
From (ii) and (iv), we get, H G > K H > K . be.......Ghanshyamdas .......of the third para of the
passage.
Hence I is true.
75. (d) Refer to the sentence, He gave him........delicious
From (i) and (iii), we get, H = E F H F . Hence fruits........of the third para of the passage.
II is true. 76. (e) Refer to the sentence I shall creep........bite her........
58. (a) P Q ...(i) , N = M ....(ii), M > R ...(iii) , R P ...(iv) hand on her forehead of the fourth para of the passage.
77. (d) Refer to the sentence, A man brought .......prince who
From (ii), (iii) and (iv), we get, N = M > R P
is missing........of the third para of the passage.
N > P or P < N . Hence I is true. 78. (c) Refer to the sentence He at once........pieces of gold
From (ii), (iv) and (i), we get, M > R P Q No of the second last sentence of fourth para of the
passage.
conclusion about the relationship between M and Q
79. (b) Refer to the sentence Finally, the king
can be established.
declared........handsomely rewarded of the fourth para
61. (d) Other groups consist of consecutive elements.
of the passage.
62. (a) 2 + 22 + 8 + 5 + 6 + 0 + 9 + 92 + 0 + 3 = 118 80. (b) The moral of the story can possibly be A good deed
Where value of alphabet = 0; symbol = square of the never goes in vain.
previous number; number = the value itself. 81. (d) It should be here was her sons future.
63. (b) The letters used in the series are B Q E R D U K A P. 82. (c) The correct spelling should be triumphant.
Sum of the positions of consonants according to 83. (a) The word suppressed should be replaced by revealed
alphabet = 2 + 17 +18 + 4 + 11 + 16 = 68. Similarly, sum or leaked in the sentence.
of the vowels positions = 5 + 21 + 1 = 27. Hence required 84. (b) It should be handed over which means the act of
difference = 68 27 = 41. moving power or responsibility from one person to
64. (e) First element of each group follows the order + 3, + 3, another.
+3... and so on. 85. (a) The correct spelling is difficult.

You might also like